Você está na página 1de 89

Aliando Menelaus e Lei dos Senos

1.

Teorema de Menelaus

Este e um dos mais importantes teoremas da geometria, pois transforma problemas de colinearidade em
verificar que um produto de razoes e 1.
Teorema de Menelaus. Sejam P , Q e R pontos sobre os lados AB, BC e CA (ou seus prolongamentos)
do triangulo ABC, respectivamente. Os pontos P , Q e R sao colineares se, e somente se,
P A QB RC

=1
P B QC RA
Ou seja, P , Q e R sao colineares se vale a conta acima, e essa conta pode ser obtida com semelhancas
ou lei dos senos.
Por outro lado, se P , Q e R sao colineares entao o teorema de Menelaus nos da uma relacao que pode
ser u
til.
Demonstra
c
ao
Provaremos primeiro que se P , Q e R sao colineares entao vale a relacao.
Sejam A0 , B 0 e C 0 as projecoes ortogonais de A, B e C na reta P QR.

Note que AA0 , BB 0 e CC 0 sao paralelas, logo os pares de triangulos P AA0 , P BB 0 , QBB 0 , QCC 0 e
RCC 0 , RAA0 sao semelhantes. Portanto, das semelhancas,
PA
AA0
=
,
PB
BB 0

QB
BB 0
=
,
QC
CC 0

RC
CC 0
=
RA
AA0

Multiplicando as tres razoes acima obtemos a relacao desejada.


Agora provemos a recproca. Suponha, por absurdo, que valha a relacao
P A QB RC

=1
P B QC RA
mas que P , Q e R nao sao colineares.

Seja entao R0 a intersecao de P Q e AC. Entao, como acabamos de provar,


P A QB R0 C

=1
P B QC R0 A
Comparando as duas u
ltimas equacoes, obtemos
RC
R0 C
= 0 ,
RA
RA
ou seja, R e R0 dividem AC na mesma razao. Logo R = R0 .
O leitor deve verificar a veracidade do fato para outras posicoes da reta em relacao ao triangulo.
2.

Um problema

Problema (Lista 4, prepara


c
ao para a IMO 2006). Seja ABCD um quadrilatero convexo tal que AB
e CD encontram-se em P e AD e BC encontram-se em Q. Se O e um ponto no interior de ABCD tal que
6 BOP = 6 DOQ, prove que 6 AOB + 6 COD = 180 .
Resolu
c
ao
Primeiro provaremos o seguinte lema, que pode ser u
til para varios problemas, considerando a figura tao
freq
uente.
Lema. Sejam AB, CD, AD e BC retas tais que AB e CD se cortam em P e AD e BC se cortam em Q.
Entao
QA QC
PA PC
=
PB PD
QB QD
Demonstra
c
ao

Aplicando o teorema de Menelaus a` reta P AB e ao triangulo QCD, obtemos


P C AD BQ

=1
P D AQ BC
Aplicando o teorema de Menelaus agora a` reta P CD ao triangulo QAB, obtemos
P A CB DQ

=1
P B CQ DA

Multiplicando as duas equacoes, obtemos


P C AD BQ P A CB DQ
PA PC
QA QC

= 1
=
P D AQ BC P B CQ DA
PB PD
QB QD

Observe que, como o teorema de Menelaus vale independentemente da posicao entre a reta e o triangulo,
esse lema tambem vale independentemente da posicao de A, B, C e D.
Esse lema e u
til porque serve como um substituto para a potencia de ponto. A diferenca e que esse
lema vale mesmo se ABCD nao for um quadrilatero inscritvel.
Voce tambem pode demonstrar esse lema diretamente com a lei dos senos. Tente!
Voltemos ao problema. Note que 6 BOP = 6 DOQ e equivalente a 6 BOQ = 6 DOP .

Devemos provar uma relacao entre angulos a partir de uma outra igualdade de angulos, mas isso nao
parece facil, considerando que na figura ha muitos outros angulos, a maioria diferentes entre si. Assim,
parece muito logico termos que utilizar a lei dos senos ou o teorema de Menelaus ou. . . ambos! Ao utilizar a
lei dos senos, e importante ter algum ponto de partida, ou seja, alguma relacao entre medidas para comecar,
para que tudo pssoa se cortar e obtermos uma equacao trigonometrica. O lema acima nos prove essa relacao:
PA PC
QA QC
=
PB PD
QB QD
Agora temos que relacionar tudo isso com o ponto O, que e o vertice dos angulos envolvidos na igualdade
que queremos provar. Da-lhe lei dos senos! E note ainda que os segmentos mais interessantes para fazer as
contas sao OA, OB, OC e OD. Vamos, entao, a` lei dos senos. Primeiro, com o ponto P :
PA
=
sen 6 P OA
6

OA
AP O

PB
=
sen 6 P OB
6

OB
BP O

PC
=
sen 6 P OC
6

OC
CP O

Observando que 6 AP O = 6 BP O e 6 CP O = 6 DP O, obtemos


PA PC
OA OC sen 6 P OA sen 6 P OC
=

PB PD
OB OD sen 6 P OB sen 6 P OD
Analogamente,

Portanto, do lema,

QA QC
OA OC sen 6 QOA sen 6 QOC
=

QB QD
OB OD sen 6 QOB sen 6 QOD
sen 6 QOA sen 6 QOC
sen 6 P OA sen 6 P OC
=
sen 6 P OB sen 6 P OD
sen 6 QOB sen 6 QOD

PD
=
sen 6 P OD
6

OD
DP O

Note que ate agora nao utilizamos o fato de que 6 BOP = 6 DOQ, ou seja, esse fato pode ser utilizado
em outros problemas (mas se cair numa prova, demonstre-o!).
Agora vamos utilizar a igualdade de angulos dada no enunciado. De 6 BOP =
DOP , obtemos
sen 6 P OA sen 6 P OC = sen 6 QOA sen 6 QOC
6

DOQ e
6

BOQ =

Utilizando o bom e velho prostaferese, obtemos


cos(6 P OA + 6 P OC) cos(6 P OA 6 P OC) = cos(6 QOA + 6 QOC) cos(6 QOA 6 QOC)

Mas 6 P OA + 6 P OC + 6 QOA + 6 QOC = 360 (veja a figura!), logo os co-senos de 6 P OA + 6 P OC e


QOA + 6 QOC sao iguais. Portanto
cos(6 P OA 6 P OC) = cos(6 QOA 6 QOC)
Considerando que cada diferenca e maior ou igual a 0 e menor do que 180 , temos
6

P OA 6 P OC = 6 QOA 6 QOC ou 6 P OA 6 P OC = 6 QOC 6 QOA

Tendo em mente que 6 P OA + 6 P OC + 6 QOA + 6 QOC = 360 , a primeira equacao e equivalente a


P OC+6 QOA = 180 e a segunda equacao e equivalente a 6 P OA+6 QOA = 180 . Mas essa u
ltima equivale
a dizer que o angulo 6 P OQ e 180 , o que nao e possvel pois O pertence ao interior do quadrilatero ABCD.
Logo 6 P OC + 6 QOA = 180 (6 P OC 6 DOP ) + (6 QOA + 6 BOQ) = 180 6 COD + 6 AOB =
180 .
6

Conjugados isogonais e simedianas


1.

Conjugados isogonais

A ideia de conjugado e fazer uma associacao entre objetos. Objetos conjugados supostamente tem propriedades semelhantes. Isso e bastante comum em equacoes: se um n
umero e raiz, ent
ao o conjugado
tambem e raiz. Em geometria, tambem existe a ideia de conjugado. De fato, dado um triangulo, cada ponto
tem um conjugado isogonal e um conjugado isotomico. Aqui, trataremos somente de conjugados isogonais.
Defini
c
ao 1.1. Dado um triangulo ABC, o conjugado isogonal em relacao a ABC de um ponto T do
plano de ABC e obtido refletindo as retas T A, T B e T C em relacao `as bissetrizes internas de ABC que
passam por A, B e C, respectivamente. As retas resultantes sao concorrentes no isogonal T 1 de T .
A seguir, as linhas pontilhadas sao as bissetrizes, e as cevianas cinzas sao as reflex
oes das cevianas
pretas.
A

T -1

T
B

O fato de que as retas isogonais sao concorrentes e extremamente importante, tanto que sera enunciado
novamente.
Teorema fundamental dos conjugados isogonais. Dados um triangulo e tres retas que passam pelos
respectivos vertices e concorrem em um ponto P , as retas isogonais a elas, obtidas atraves da reflex
ao em
relacao `a bissetriz interna correspondente, sao concorrentes no conjugado isogonal P 1 de P .
Demonstra
c
ao
Por que as cevianas cinzas sao concorrentes? Isso decorre de duas aplicacoes do teorema de Ceva trigonometrico: primeiro com as cevianas concorrentes em T e depois, com as cevianas concorrentes em T 1 , que
formam os mesmos
angulos que as outras cevianas, porem no sentido contrario.
Na verdade, pode ocorrer de as tres cevianas serem paralelas. Isso ocorre se, e somente se, T est
a sobre
o circuncrculo de ABC; nesse caso, pensamos projetivamente, ou seja, o conjugado isogonal e um ponto do
infinito.
1.1.

Para que servem isogonais?

O que e mais u
til em conjugados isogonais e simplesmente que as cevianas sao reflex
oes umas das outras em
relacao `as bissetrizes, e isso costumam levar a algumas igualdades entre angulos um pouco mais difceis de
obter ou mesmo de se imaginar com contas.

Exemplo 1.1.
No triangulo ABC, P e Q sao pontos no interior de ABC tais que 6 CBP = 6 P BQ = 6 QBA = 6 ABC/3
e 6 BCP = 6 P CQ = 6 QCA = 6 ACB/3. Sejam D e E as projecoes ortogonais de P sobre AB e AC,
respectivamente. Prove que AQ e perpendicular a DE.
Resolu
c
ao
Seja = 6 P AD. Ent
ao 6 AP D = 90 e, como
6
inscritvel. Logo AED = 6 AP D = 90 .
A
6

ADP e
6

AEP sao retos, o quadril


atero ADP E e

q
q
Q

90 - q

90 - q
a

b
b
b

Olhando a figura, note que basta provarmos que 6 QAC = . A e que entram os conjugados isogonais.
Como 6 P BC = 6 QBA e 6 BCP = 6 QCA, os pares de retas BP ; BQ e CP ; CQ sao simetricos entre si
em relacao `as bissetrizes de 6 ABC e 6 ACB, respectivamente. Ou seja, P e Q sao conjugados isogonais
e, portanto, 6 P AB e 6 QAC tambem sao iguais. Logo 6 QAC = e o angulo entre as retas AQ e DE e
180 (90 ) = 90 .
Note que para provar o resultado na conta, bastaria repetir a demonstracao do teorema fundamental
dos conjugados isogonais. Mas o mais interessante e que, sabendo da existencia dos conjugados isogonais, e
natural pensar nessa solucao. Em contraste, fazer a conta sem pensar em conjugados isogonais n
ao parece
ser t
ao natural assim. Ent
ao d
a para pensar que os conjugados isogonais nos economizou n
ao so fazer a
conta, mas mostrou onde fazer as contas relevantes.
1.2.

Conjugados isogonais dos pontos not


aveis

Voce ja deve estar familirializado com os pontos notaveis do triangulo: o baricentro (encontro das medianas),
o incentro (encontro das bissetrizes internas), o ortocentro (encontro das alturas) e o circuncentro (encontro
das mediatrizes). Quais sao os conjugados isogonais desses pontos? Vamos aproveitar e conhecer mais um
ponto notavel (mas n
ao t
ao conhecido).
Vamos fazer isso em ordem de dificuldade.
Incentro
As projecoes coincidem com as proprias bissetrizes. Logo o conjugado isogonal do incentro, que e o
encontro das bissetrizes internas, e ele mesmo.
O mesmo vale para os ex-incentros (encontros de duas bissetrizes externas e uma bissetriz interna e
centros dos ex-incrculos, que sao tangentes externamente aos lados ou seus prolongamentos). Pense sobre o
assunto!

Ortocentro e circuncentro
A figura a seguir deve convence-lo de que o ortocentro e o circuncentro sao conjugados isogonais.

Baricentro
Os isogonais das medianas sao as simedianas (SImetrico + MEDIANA). O ponto de encontro das
simedianas e o ponto de Lemoine, tambem conhecido como ponto simediano. O ponto de Lemoine e costumeiramente denotado por K.
Primeiro, vamos aprender a traca-las de modo mais pratico.
Lema. Sejam D a intersecao das retas tangentes ao circuncrculo do triangulo ABC por B e C. Ent
ao a
reta AD contem a simediana que passa por A.
Demonstra
c
ao

Construa o paralelogramo ABEC. Ent


ao AD contem a mediana AM . Afirmamos que D e E sao
conjugados isogonais. De fato, 6 BCE = 6 B e o angulo entre AC e CD, pela tangencia, e igual a 6 B.
Assim, as retas CD e CE sao conjugadas isogonais. Analogamente, BD e BE tambem sao, e o resultado
segue do teorema fundamental dos conjugados isogonais.
2.

Tri
angulo pedal

Defini
c
ao 2.1. Seja P um ponto no plano do triangulo ABC e D, E e F as projecoes de P sobre as retas
BC, CA e AB. O triangulo DEF e o triangulo pedal de P em relacao ao triangulo ABC.
O que triangulos pedais tem de especial? Primeiro, aparecem muitos angulos retos, o que propicia o
aparecimento de quadril
ateros inscritveis. Segundo, eles normalmente minimizam areas.
Teorema do mnimo. Dado um triangulo T , considere todos os triangulos DEF semelhantes a T , todos
na mesma ordem, com D sobre o lado BC, E sobre o lado CA e F sobre o lado AB. Dentre todos esses
triangulos, o de menor
area e o triangulo pedal de algum ponto P .
Demonstra
c
ao
N
ao provaremos aqui a existencia de um triangulo de area mnima (caso voce esteja curioso, estude topologia
e depois volte!).
Seja DEF o triangulo de
area mnima. Seja M o ponto de Miquel de ABC e DEF , e sejam P , Q e R
as projecoes de M sobre os lados.

Note que o quadril


atero CP M Q e inscritvel (pois 6 M P C e 6 M QC sao retos), de modo que 6 DM E =
P M Q = 180 6 C. Portanto, 6 P M D = 6 QM E: imagine o angulo 6 DM E girando em torno de M para
coincidir com 6 P M Q; M D vira M P e M E vira M Q. Analogamente, 6 RM F = 6 QM E.
6

Portanto os triangulos P M D, QM E e RM F sao semelhantes e induzem uma roto-homotetia (voce se


MP
lembra o que e isso?) que leva DEF a P QR. A raz
ao de homotetia e MD
1, de modo que a area de P QR
e menor ou igual `
a
area de DEF . Como DEF tem area mnima, os triangulos devem ser congruentes e
deste modo M P = M D, ou seja, P = D. Analogamente, Q = E e R = F , de modo que DEF e o triangulo
pedal de P .

Exemplo 2.1.
(Prova de Selecao EUA, 2008) Sejam P , Q, R pontos sobre os lados BC, CA, AB de um triangulo acut
angulo
ABC tais que P QR e equilatero e tem
area mnima entre todos tais triangulos equilateros. Prove que a reta
perpendicular a QR que passa por A, a reta perpendicular a RP que passa por B e a reta perpendicular a
P Q que passa por C tem um ponto comum.
Resolu
c
ao
Pelo teorema do mnimo, P QR e triangulo pedal de algum ponto T .

Como os
angulos 6 T QA e 6 T RA sao ambos retos, o quadril
atero AQT R e inscritvel, e o seu circuncentro
e o ponto medio A de AT . Assim, a reta perpendicular a QR e que passa por A, que contem a altura relativa
a QR, e isogonal a AT , que contem o circuncentro, em relacao ao triangulo AQR. Como os angulos 6 BAC
e 6 QAR sao iguais, a perpendicular e AT sao isogonais em relacao ao triangulo ABC tambem. O analogo
para as perpendiculares a P R por B e a P Q por C. Como AT , BT e CT sao concorrentes em T , seus
isogonais sao concorrentes no conjugado isogonal de T .
A ttulo de curiosidade, o ponto T e o primeiro ponto isodinamico. Os dois pontos isodinamicos (adivinhe
o nome do outro ponto!) sao os pontos de intersecao dos crculos de Apolonio de A, B e C (que passam
pelos vertices, o pe da bissetriz interna e tem centro sobre o lado oposto). Os seus conjugados isogonais sao
os pontos de Fermat. O primeiro ponto de Fermat e o ponto cuja soma das distancias aos vertices e mnima
(supondo que os
angulos internos do triangulos sao todos menores do que 120 ). Veja [5] para aprender isso
e muito, muito mais.
2.1.

Voltando `
as simedianas

Uma aplicacao interessante da ideia de triangulo pedal est


a relacionado `as simedianas. Uma outra maneira
de construir as simedianas e a seguinte:

Lema. Construa quadrados ABBc Ac , BCCa Ba e CAAb Cb externamente sobre os lados do triangulo ABC.
Prolongue Ac Bc , Ba Ca e Cb Ab para obter o triangulo A B C . Ent
ao as retas AA , BB e CC concorrem

no ponto simediano K de ABC.

Demonstra
c
ao
Por simplicidade, sejam BC = a, CA = b e AB = c e L o encontro de AA , BB , CC . Queremos provar
que L = K.
Primeiro, como os pares de retas AB; A B , BC; B C e CA; C A sao paralelos, os triangulos ABC e
A B C sao semelhantes. Seja k a raz
ao de semelhanca. Sejam ka , kb e kc as distancias de K a BC, CA e
AB, respectivamente. Das semelhancas entre KAB; KA B , KBC; KB C e KCA; KC A , todas de raz
ao
k,
kb
kc
ka
kb
kc
k
ka
=
=
= k
=
=
=
ka + a
kb + b
kc + c
a
b
c
1k

Isto quer dizer que as distancias de L a cada um dos lados e proporcional aos seus comprimentos. Alem
disso, considerando uma semelhanca prova-se que um ponto X pertence a, digamos, AL se, e somente, as
distancias de X aos lados AB e AC sao proporcionais a seus comprimentos. Basta provar que a simediana
por A tem a mesma propriedade. Para isso, considere a construcao anterior, sendo D o mesmo ponto definido
anteriormente.

Sendo x e y as distancias de D a AB e AC, respectivamente, considerando que o angulo entre AB e


BD e 6 F BD = 6 ACB = 6 C e o
angulo entre AC e CD e 6 DCE = 6 ABC = 6 B (nao se preocupe com

triangulos obtusangulo; nesse caso, troque o angulo obtuso por seu suplementar), nos triangulos ret
angulos
BDF e CDE, x = BD sen 6 C e y = DC sen 6 B. Observando ainda que, sendo DB e DC tangentes,
sen 6 C
AB
DB = DC, temos xy = sen
em. Da mesma
6 B = AC . Logo D pertence a AL e, consequentemente, K tamb
forma provamos que K pertence a BL e CL, de modo que L = K.
Assim como no teorema das bissetrizes, as simedianas dividem os lados opostos em raz
oes interessantes.
Lema. Seja ABC um triangulo e AN uma simediana. Ent
ao

BN
CN


AB 2
.
AC

Demonstra
c
ao

Ja provamos anteriormente que as distancias do ponto simediano K aos lados sao proporcionais a seus
comprimentos. Ent
ao existe t real tal que ka = ta, kb = tb e kc = tc. Assim, as areas de KAB, KAC e
tc2 tbb
tb2
taa
ta2
KBC sao tcc
=
2
2 , 2 = 2 e 2 = 2 , respectivamente. Logo
BN
area ABN

area KBN

area ABN area KBN


area KAB
c2
=
=
=
=
= 2 =
CN
rea ACN
a
rea KCN
a
area ACN area KCN
area KAC
b

AB
AC

2

Isso pode ser generalizado:


Lema. Sejam da , db e dc as distancias de um ponto P aos lados BC, CA e AB do triangulo ABC. Se AP
cdc
corta BC em N , ent
ao BN
CN = bdb .
Demonstra
c
ao
Fica a cargo do leitor.
Exerccios
01. Sejam BD e CE alturas do triangulo ABC. As retas tangentes ao circuncrculo de ABC em B e C
cortam-se em T . Prove que a reta AT bissecta DE.
02. (Ibero 2007, Problema 2) Seja ABC um triangulo de incentro I. O crculo tem centro em I, raio
maior do que o inraio de ABC e n
ao passa por nenhum vertice de ABC. Seja X1 a intersecao de com AB
mais proxima de B; X2 e X3 as intersecoes de com BC, com X2 mais proximo de B; X4 a intersecao de
com AC mais proxima de C. Finalmente, seja K a intersecao de X1 X2 e X3 X4 . Prove que AK bissecta
o segmento X2 X3 .
03. Sejam P e Q pontos no interior do
angulo 6 BAC tais que BP = CP , BQ = CQ e 6 ABP + 6 ACQ =

6
6
180 . Prove that BAP = CAQ.
04. As retas obtidas atraves das reflex
oes da diagonal BD do quadril
atero ABCD em relacao `as bissetrizes
oes da diagonal AC do quadril
atero ABCD
de 6 B e 6 D passam pelo ponto medio de AC. Prove que as reflex
em relacao `as bissetrizes de 6 A e 6 C passam pelo ponto medio de BD.

05. (Prova de Selecao EUA, 2008) Seja ABC um triangulo e G o seu baricentro. O ponto P varia sobre
o segmento BC. Os pontos Q e R pertencem aos lados AC e AB respectivamente, e sao tais que P Q e
paralelo a AB e P R e paralelo a AC. Prove que, ao variar P sobre BC, o circuncrculo de AQR passa por
um ponto fixado X tal que 6 BAG = 6 CAX.
6

06. (IMO 2004, Problema 5) Num quadril


atero convexo ABCD a diagonal BD n
ao e bissetriz do angulo
ABC nem do
angulo 6 CDA. Um ponto P no interior de ABCD satisfaz
6

P BC = 6 DBA

e
6

P DC = 6 BDA.

Prove que os vertices do quadril


atero ABCD pertencem a uma mesma circunferencia se, e somente se,
AP = CP .
07. (USAMO 2008, Problema 2) Seja ABC um triangulo acut
angulo e escaleno. Os pontos M , N , P sao
os pontos medios de BC, CA e AB, respectivamente. As mediatrizes de AB e AC cortam a semirreta AM
em D e E, respectivamente. As retas BD e CE cortam-se em F , que e interior ao triangulo ABC. Prove
que os pontos A, N , F , P est
ao sobre uma mesma circunferencia.
08. (Cone Sul 2009, Problema 3) Sejam A, B e C tres pontos tais que B e ponto medio do segmento AC
ao construdos o triangulo equilatero P CQ tal que B e Q est
ao
e seja P um ponto tal que 6 P BC = 60 . S
em semiplanos diferentes em relacao a P C, e o triangulo equilatero AP R tal que B e R est
ao no mesmo
semiplano em relacao a AP . Seja X o ponto de intersecao das retas BQ e P C; seja Y o ponto de intersecao
das retas BR e AP . Demonstre que XY e AC sao paralelos.
3.

Refer
encias Bibliogr
aficas

[1] Uma otima fonte de problemas e o Mathlinks: http://www.mathlinks.ro/ (em ingles).


[2] O livro Modern Geometry of the Triangle, de William Gallatly, contem muita informacao interessante,
incluindo a maior parte dos fatos sobre simedianas e o ponto simediano.
[3] Mais conjugados isogonais? Isso e muito mais no livro Geometry of Conics (o livro do bode veja a
capa!), de A. V. Akopyan e A. A. Zaslavsky.

Corte, deslize, compare e conclua


Alguns problemas de Geometria podem ser resolvidos com. . . tesoura, cola e xerox! Numa prova, talvez estes
n
ao sejam os instrumentos mais adequados; ent
ao vamos recortar, colar, ampliar e reduzir no papel mesmo
e tentar enxergar o que acontece sem realmente fazermos essas operacoes.
N
ao poderamos deixar de lado o teorema mais famoso da Matem
atica.
1.

Duas demonstra
co
es do teorema de Pit
agoras

Todos conhecem o teorema de Pit


agoras.
Teorema 1.1. (Teorema de Pit
agoras) Num triangulo ret
angulo, o quadrado da hipotenusa e igual `a soma
dos quadrados dos catetos.
Vamos demonstrar o teorema de duas maneiras: uma na forma de um poema em ingles e outra recortando
e colando. . . uma rosquinha!
Primeira demonstra
c
ao.
A demonstracao e o poema a seguir e de George Biddle Airy, que foi astronomo real britanico de 1836 a
1881.

Here I am, as you may see,


a 2 + b2 - ab
When two triangles on me stand,
Square of hypothn is plannd;
But if I stand on them instead,
The square of both the sides
[are read.

Traducao:
Aqui, estou, como voce pode ver,
a2 + b2 ab
Quando dois quadrados em mim se apoiam,
O quadrado da hipotenusa e evidente;
Mas se sou eu quem neles se apoia
Os quadrados de ambos os lados sao lidos.

Segunda demonstra
c
ao.
A figura a seguir mostra como recortar dois quadrados de um quadrado so.

O triangulo ret
angulo em quest
ao e o que tem como hipotesuna o lado esquerdo do quadrado.
Vamos demonstrar que os polgonos que temos realmente sao dois quadrados. Primeiro, note que os
dois triangulos ret
angulos com hipotenusas iguais ao lado esquerdo e ao lado superior do quadrado sao
congruentes. Ent
ao, considerando os catetos correspondentes, os dois polgonos mostrados na figura da
direta sao quadrados com lados iguais aos catetos do triangulo ret
angulo.
Mas o que e mais interessante foi o que motivou essa demonstracao. Um problema muito interessante e
tentar cortar um quadrado em varios quadrados menores, todos de tamanhos diferentes (se os lados pudessem
ser iguais, teramos um problema nada interessante!). S
o que podemos estender o problema: por exemplo,
como cobrir a superfcie de um cubo com quadrados de lados diferentes? E a de uma rosquinha?
Primeiro, temos que entender como construir um toro (a rosquinha) a partir de um quadrado (feito de
material elastico). Primeiro, colamos os lados opostos do quadrado, obtendo um cilindro. Depois, colamos
as bordas de um cilindro, obtendo o toro.

Agora, suponha que exista um planeta muito, muito distante com a forma de um toro (como seria viver
numa rosquinha? Um dia tenho que perguntar isso `as formigas!). Esse planeta, assim como o planeta Terra,

pode ter um mapa. Como seria esse mapa? A propria maneira como construmos o toro nos responde: seria
um quadrado! Os dois lados horizontais representam os mesmos lugares, assim como os dois lados verticais.
Note a diferenca com os mapas da Terra: o mapa da a volta pela horizontal, mas n
ao pela vertical (a n
ao
ser que voce acredite que o P
olo Norte e o P
olo Sul sejam o mesmo lugar!).

Calma! A pessoa e o cachorro esto inteiros!

Suponha que nesse planeta existam dois pases, a Quadradaol


andia e a Rep
ublica Quadradinhense.
Ambos os pases orgulham-se de sua forma quadrada. Olhe a primeira figura dessa demonstracao: esse e
o mapa desse planeta! A Rep
ublica Quadradinhense est
a em cinza e a Quadradaol
andia, em branco. Com
isso, conseguimos cobrir o toro com dois quadrados de lados diferentes. Incrvel, n
ao?
2.

Paralelogramos e tri
angulos ret
angulos

Paralelogramos sao quadril


ateros cujos lados opostos s
ao paralelos. Um teorema bastante importante e
Teorema 2.1.

As diagonais de um paralelogramo cortam-se em seus respectivos pontos medios.

Demonstra
c
ao
Uma maneira de obter um paralelogramo a partir de um triangulo e duplic
a-lo e gira-lo 180 em torno do
ponto medio de um de seus lados. Os
angulos marcados asseguram o paralelismo dos lados opostos.

E se, antes de girarmos o triangulo, ligarmos o ponto medio ao vertice? Esse segmento, cujo nome e

mediana, tambem gira 180 em torno do ponto medio:

Mas ao girarmos um segmento 180 em torno de uma de suas extremidades e unirmos ao segmento antes
de girado, obtemos um segmento com o dobro do tamanho. Logo o ponto medio do lado tambem e ponto
medio do segmento.
Uma aplicacao desse importante fato e outro importante teorema:
Teorema 2.2.
sao iguais.

As tres distancias do ponto medio da hipotenusa de um tri


angulo ret
angulo a seus vertices

Demonstra
c
ao
Um triangulo ret
angulo pode ser entendido como a metade de um ret
angulo. Assim, e so lembrar que um
ret
angulo e um paralelogramo com diagonais congruentes e observar a figura.

3.

O baricentro

o encontro das medianas de um triangulo.


O baricentro e um dos pontos not
aveis do triangulo. E
Como temos tres medianas, n
ao e
obvio que elas passam por um mesmo ponto. Isso deve ser demonstrado.
Teorema 3.1. As tres medianas de um triangulo passam por um mesmo ponto, chamado baricentro do
triangulo. Alem disso, o baricentro divide cada mediana na raz
ao 2 : 1.
Demonstra
c
ao
Demonstraremos que o encontro G de duas medianas divide ambas na raz
ao 2 : 1. Considere um triangulo
T e desenhe duas medianas ma e mb . Em seguida, faca tres c
opias de T : em uma, Ta , copie tambem ma ,

mas somente ma ; em outra, Tb , copie mb , mas somente mb ; e na outra, T , n


ao copie mais nada. Com T e
suas tres c
opias, monte um triangulo M maior, como mostra a figura.

Ta

Tb
T

ma

mb

Observe que o quadril


atero formado por T e Ta e um paralelogramo. Logo uma das diagonais contem ma .
O mesmo vale para o paralelogramo formado por T e Tb e mb . Portanto essas diagonais dos paralelogramos
sao medianas do triangulo maior M . Ou seja, as medianas de M contem as medianas de T . Note que o
triangulo maior e uma c
opia ampliada de T ; isso e verdade porque todas as suas dimensoes sao o dobro das
dimensoes correspondentes de T . Em outras palavras, M e T sao semelhantes, com raz
ao de semelhanca
igual a 2.
Agora, desenhe T mas, em vez de ma e mb , desenhe esses segmentos desde os vertices do triangulo ate
o ponto de intersecao. Faca uma c
opia M de T duas vezes maior. Encaixe esses dois triangulos como na
figura anterior, obtendo

2x

G
T

Note que o pedaco da mediana de T e o pedaco correspondente na c


opia ampliada, duas vezes maior,
unidas por G, formam a mediana do triangulo M . Logo, em todo triangulo M , a intersecao entre duas
medianas as cortam na raz
ao 2 : 1.
A partir desse fato, se considerarmos a intersecao G de uma dessas duas medianas com a terceira
mediana, ela dividira tambem ambas na raz
ao 2 : 1. Em particular, essa intersecao divide a primeira
mediana na raz
ao 2 : 1, ent
ao G = G e o teorema segue.
4.

Um problema da IMO 2005

Todos os teoremas que demonstramos sao conhecidos. Mas alguns problemas de Olimpada podem ser
resolvidos com as ideias que vimos. Em particular, o seguinte problema:
Problema 1, IMO 2005. Escolhemos seis pontos sobre os lados do triangulo equilatero ABC: A1 , A2
sobre BC; B1 , B2 sobre AC; C1 , C2 sobre AB. Essa escolha e feita de modo que A1 A2 B1 B2 C1 C2 e um
hex
agono convexo com todos os seus lados iguais. Prove que A1 B2 , B1 C2 e C1 A2 sao concorrentes.
Resolu
c
ao
Ao desenhar o hex
agono, podemos pensar em recorta-lo! Se tirarmos o hex
agono e rearranjarmos a figura,
obtemos
A

B2
x

x
x
C1

x
x

B1

x
C2

x
x

A1

A2

Ou seja, obtemos um triangulo equilatero dentro do outro. Mas sera que as pecas se encaixam direitinho
so deslizar cada um dos triangulos C2 A1 B e A2 B1 C ao longo de AB e AC, respectivamente.
mesmo? Sim! E
Cada lado diminui de x, ent
ao encaixam-se perfeitamente num triangulo equilatero.
Vamos calcular alguns
angulos. Considere o vertice P na figura abaixo e sejam , e as medidas dos
angulos em torno desse vertice. Temos + 60 + = 180.
o

60
a
x

P
o

60

b
x

60

60

Observando os dois triangulos cinzas com P como um de seus vertices, nota-se que no triangulo superior
est
ao marcados os
angulos internos de medidas e 60 , ou seja, o outro angulo mede ; no triangulo da

direita est
ao marcados os
angulos internos de medidas e 60 , sendo que o outro angulo mede . Isso quer
dizer que os triangulos cinzas sao todos congruentes!
Voltemos agora `
a figura original e recorte tres triangulos do hex
agono, ou melhor, trace tres de suas
diagonais. Pela congruencia entre os triangulos AB2 C1 , BC2 A1 e CA2 B1 , os angulos externos 6 C1 B2 B1 ,
6 B1 A2 A1 e 6 A1 C2 C1 s
ao congruentes. Portanto, pelo caso LAL, os triangulos C1 B2 B1 , B1 A2 A1 e A1 C2 C1
sao congruentes e, portanto, B1 C1 = A1 B1 = C1 A1 , ou seja, o triangulo A1 B1 C1 e equilatero!

B2
x

C1
B1

x
C2

x
q

x
B

A1

A2

Para terminar, note que como A1 B1 = A1 C1 e B2 B1 = B2 C1 , a reta A1 B2 e mediatriz de B1 C1 .


Analogamente, A2 C1 e mediatriz de A1 B1 e C2 B1 e mediatriz de A1 C1 . Como as mediatrizes de um
triangulo sao concorrentes, o resultado segue.
Exerccios
01. No hex
agono ABCDEF , lados opostos sao congruentes e paralelos. Prove que os triangulos ACE e
BDF tem a mesma
area.
02. Prove que se a e b sao dois lados de um triangulo, ent
ao a sua area n
ao excede ab/2.
03. Seja Q um quadril
atero cujoa lados medem a, b, c, d, nessa ordem. Prove que a area de Q n
ao excede
(ac + bd)/2.
04. Seja ABC um triangulo e M , N e P os pontos medios de BC, CA e AB, respectivamente. Prove que
o triangulo cujos lados tem medidas iguais `as das medianas AM , BN e CP tem area igual a 3/4 da area de
ABC.
05. Na u
ltima secao, utilizamos um fato que n
ao demonstramos por ser bem conhecido: as mediatrizes dos
lados de um triangulo cortam-se num mesmo ponto. Vamos prova-lo neste exerccio. A mediatriz de um
segmento e a reta perpendicular a ele que passa pelo seu ponto medio.
(a) Prove que todo ponto P da mediatriz de um segmento AB est
a `a mesma distancia ate A e B, ou seja,
que P A = P B. Dica: desenhe o segmento AB num papel e dobre-o de modo que A coincida com B;
desdobre e escolha um ponto P sobre a dobra e trace P A e P B; dobre de volta. O que acontece?
(b) Prove que as tres mediatrizes de um triangulo se encontram em um mesmo ponto. Esse ponto e chamado
circuncentro. Dica: e menos difcil do que parece! Considere o encontro O entre duas mediatrizes do
triangulo ABC; qual a relacao entre OA, OB e OC?
06. Prove que as alturas (ou seus prolongamentos) de um triangulo passam por um mesmo ponto. Esse
ponto e chamado ortocentro de ABC. Dica: Seja ABC um triangulo e M , N e P os pontos medios de BC,
CA e AB, respectivamente. Os que sao as mediatrizes de ABC para M N P ?

07. (Reta de Euler) Esse e um dos resultados mais interessantes da Geometria:


Teorema 4.1. (Reta de Euler). Em todo triangulo, o ortocentro H, o baricentro G e o circuncentro est
ao
alinhados, nessa ordem. Alem disso, HG = 2 GO.
Prove esse teorema.
Dica: observe a figura a seguir, sendo M , N e P os pontos medios de BC, CA e AB, respectivamente.
Considere a c
opia M N P O reduzida de ABCH. Qual a raz
ao entre AH e M O? Voce consegue enxergar
alguma semelhanca envolvendo O, G e H?

5.

Refer
encias bibliogr
aficas

[1] Ian Stewart, Mania de Matem


atica. O ttulo original em ingles e Math Hysteria (Fun and Games
with Mathematics). Ambas as demonstracoes do teorema de Pit
agoras foram extradas desse livro,
que e um livro muito divertido sobre assuntos matematicamente inusitados como calend
arios, Banco
Imobili
arioTM , partilhas justas de bolo, o jogo Liga-Ponto, Campo Minado, entre outros.
[2] Kiran Kedlaya, Notes on Euclidean Geometry. Este arquivo est
a disponvel em
http://www.unl.edu/amc/a-activities/a4-for-students/problemtext/geom-080399.ps
e em
http://math.mit.edu/ kedlaya/geometryunbound/geom-080399.pdf
essencialmente um curso de geometria para olimpadas. Muitos exerccios foram retirados da primeira
E
secao desse arquivo.
[3] A demonstracao da reta de Euler e muito mais est
a no livro Geometry Revisited, de H. S. M. Coxeter
e S. L. Greitzer. Esse livro tem um
otimo curso de geometria.

As Cr
onicas de N
erdia:
o Crculo, a Rota
c
ao e os Isogonais
Quero comecar com uma discuss
ao bastante interessante.
1.

Geometria sint
etica geometria analtica

Desde a epoca de aluno (em 1995), eu sempre resolvi os problemas de Geometria com contas, em especial
trigonometria, do jeito que Edmilson e Eduardo Tengan me ensinaram e praticaram. Em 2000, comecei a
treinar alunos e acabei passando a tecnologia. A lei dos senos, para n
os, paulistas, passou a ser a sagrada
lei dos senos. Essa experiencia em fazer contas culminou no artigo Geometria com Contas, na Eureka! 17.
Por outro lado, os cearenses sempre estudaram bastante geometria sintetica, ou seja, faziam a maior
parte dos problemas com argumentos sinteticos. Ocorreu, ent
ao, uma divisao na abordagem em problemas
de Geometria: a trigonometria (geometria paulista) e a sintetica (geometria cearense). O contraste e
tamanho que a coordenacao nos problemas de Geometria na IMO 2002 foi: primeiro, os geometric people;
depois os trigonometric people.
Todavia, nesses u
ltimos anos, considerando ainda que os problemas de Geometria da IMO tem ficado
mais difceis, cheguei `
a conclusao de que e preciso saber tanto fazer contas como ter ideias sinteticas para
resolver problemas. Podemos comparar isso como estilos de luta: afinal, quem e mais poderoso: aquele que
sabe jud
o ou aquele que sabe boxe? A resposta n
ao e nem um, nem outro: e quem sabe ambos.
O proximo exemplo e os outros exerccios dessa secao servem como treinos.
Exemplo 1.1.
6

(IMO) Num triangulo ABC, seja AP a bissetriz de


ABC com Q no lado CA.
6

BAC com P no lado BC, e seja BQ a bissetriz de

Sabemos que 6 BAC = 60 e que AB + BP = AQ + QB.


Quais sao os possveis valores dos
angulos do triangulo ABC?
Resolu
c
ao
Primeiro, uma figura bem feita. O problema e que n
ao sabemos as medidas dos angulos, ent
ao a figura
certamente n
ao ficara perfeita. O maximo que podemos fazer e desenhar algumas figuras para conseguir ter
uma estimativa da resposta, se e que e u
nica.
A

30o
30o

a
B

A relacao dada AB + BP = AQ + QB e sugestiva de varias maneiras.


Solu
c
ao trigonom
etrica
A sistem
atica para se resolver problemas na conta e reduzi-los a resolver uma equacao ou provar uma
identidade trigonometrica. Aqui n
ao vai ser diferente: vamos transformar a condicao AB + BP = AQ + QB
numa equacao trigonometrica e vamos resolve-la.
Note que os triangulos que envolvem os segmentos AB, BP , AQ e QB sao ABP e AQB. Ambos tem
AB, ent
ao esse segmento deve ter um papel importante nas contas.
Reescreva a relacao dada como

BP
AQ + QB AB
=
2
2

Sendo p o semipermetro do triangulo AQB e R o circunraio do mesmo triangulo, e sendo 6 AQB/2 =


(180 ( + 60 ))/2 = 90 (30 + /2),




AQ + QB AB

sen 30 sen
= p AB = 4R cos 90 30 +
2
2
2




AB
sen
sen 30 +
=
sen( + 60 )
2
2


AB sen 2

=
2 cos 30 + 2
Acima, utilizamos a identidade p a = 4R cos

sen

 

sen

e a lei dos senos em AQB.

Assim, da relacao dada e da sagrada lei dos senos em ABP ,




sen 2
sen 2
sen 30
BP


=
=
AB
sen(30 + 2)
cos 30 + 2
cos 30 + 2

()

Agora, e so fazer as contas:





= 2 sen(30 + 2) sen
() cos 30 +
2
2





3
cos 30 +
= cos 30 +
cos 30 +
2
2





5

cos 30 +
+ cos 30 +
= cos 30 +
2
2




3
3

cos = cos 30 +
2 cos 30 +
2
2


3
1
cos 30 +
= 0 (I) ou cos =
(II)
2
2


5
2

3
2

De (II), obtemos = 60 , o que n


ao e possvel, ja que, observando os angulos internos do triangulo

ABC, 2 + 60 < 180 < 60 . Assim, vale (I) e, levando em conta que 30 < 30 + 3
2 < 120 ,
3

temos 30 + 2 = 90 = 40 .
Logo os angulos do triangulo ABC sao 6 BAC = 60 , 6 ABC = 2 = 80 e 6 BCA = 180 60 80 =

40 .
Solu
c
ao sint
etica
Relacoes como a dada no enunciado muitas vezes sugerem construcoes que as transformam em igualdades
entre segmentos. Nesse caso, temos algo muito sugestivo: AB + BP = AQ + QB pede para estendermos

AB e AQ e considerarmos P e B sobre as respectivas extensoes tais que BP = BP e QB = QB (se voce


gosta de formar imagens sobre estas construcoes, e como se BP e QB fossem portinhas com B e Q como
respectivas dobradicas; so estamos abrindo as portinhas).

P
C

Temos muito a ganhar com essas construcoes. Primeiro, por causa delas ganhamos dois triangulos
is
osceles, BP P e QBB ; e depois, temos o triangulo is
osceles: AP B e equilatero, pois 6 P AB = 60 e

AP = AB + BP = AB + BP = AQ + QB = AQ + QB = AB . E mais ainda: AP agora n


ao so e bissetriz
do nosso recem-achado triangulo equilatero, mas tambem tudo mais: altura, mediana, mediatriz, eixo de
simetria.
Ou
nico problema e que n
ao sabemos a posicao entre B e C. Pode ser que C esteja entre A e B ; pode

ser que B esteja entre A e C; e pode ate ser que B coincida com C. Temos que pensar nos tres casos.
Primeiro caso: C entre A e B

Acompanhe na figura acima: seja definido como na outra solucao. No triangulo BP P , 6 BP P = 6 BP P =


6 ABC/2 = .
Alem disso, da simetria em torno da reta AP , os triangulos AP P e AP B sao congruentes; portanto
AB P = 6 AP P = . Em particular, 6 QB P = 6 QBP = .
Enquanto isso, no triangulo QBB , 6 QB B = 6 QBB . Temos 6 QB P = 6 QBP . Subtraindo essas duas
u
ltimas igualdades membro a membro obtemos 6 P B B = 6 P BB , ou seja, o triangulo P BB e tambem
is
osceles e P B = P B .
O golpe final vem do fato de que P est
a na mediatriz de P B : por causa disso, P P = P B = P B e o

triangulo BP P e equilatero. Mas isso implica = 60 , o que e impossvel por motivos que discutimos (sem
a ajuda de senos e co-senos) na primeira solucao.
Assim, chegamos a uma contradicao e, portanto, C n
ao est
a entre A e B .

Segundo caso: B entre A e C

Q
B

P
B

O que podemos aproveitar do primeiro caso? Na verdade, tudo! Ainda temos 6 BP P = 6 BP P = ;


AP P e AP B ainda sao congruentes e, conseq
uentemente, da mesma forma, 6 QB P = 6 QBP = . Mesmo

o que fizemos no triangulo QBB e igual: obtemos 6 P B B = 6 P BB . O triangulo BP P e equilatero


de novo e chegamos num absurdo.
Por isso os dois desenhos anteriores est
ao ruins: porque n
ao sao a configuracao certa.
Deste modo conclumos que o u
nico caso valido e na verdade o
Terceiro caso: C = B

Q
B

P
P

B = C

Aqui, tudo e bem mais simples: temos 6 ACB = 6 QCB = 6 QB B = 6 QBB = 6 QBC = . No
triangulo ABC, 6 ABC + 6 ACB + 60 = 180 2 + = 120 = 40 e os angulos internos de
ABC sao 6 BAC = 60 , 6 ABC = 80 e 6 ACB = 40 .

1.1.

Qual
e a solu
c
ao mais f
acil?

Em meio a toda a discuss


ao acima, essa pergunta torna-se necessaria. Tenho um pressentimento muito forte
de que fizessemos alguma votacao, os votos se dividiriam pela metade (nao vale concentrar os votos em um
estado so!).
Tenho motivos para essa crenca: ambas as solucoes sao, de certo modo, tecnicas, no sentido que exigem
algum conhecimento e treinamento previo das ideias envolvidas. E ambas tem suas proprias dificuldades:
na solucao trigonometrica, se n
ao usassemos a identidade envolvendo p a provavelmente seria mais difcil
obter a fatoracao (e digo isso por experiencia propria; na epoca, em 2001, n
ao fiz desse jeito); por outro lado,
na solucao sintetica e importante notar a sutileza da posicao de C em relacao a B e dividir o problema em
tres casos (sendo que o u
nico caso que n
ao leva a absurdo e justamente o mais improvavel; bom, parte da
beleza da Matem
atica est
a a, n
ao?).
De qualquer forma, deixo para o leitor esse julgamento. Mas ressalto mais uma vez que saber jud
oe
boxe e mais seguro que saber jud
o ou boxe.
Alem disso, pode ocorrer de solucoes sinteticas serem mais difceis de obter que solucoes analticas e
vice-versa. Nesse caso, compare saber as tecnicas de ambas com ter itens num jogo de videogame: um item
pode ser totalmente in
util uma hora mas imprescindvel em outra; e quanto mais itens voce tiver, mais
chance tem de terminar o jogo.
Exerccios
Nos proximos problemas, tente encontrar duas solucoes: uma sintetica e outra com contas.
01. No triangulo ABC, AB = AC. D e um ponto sobre o lado BC tal que BD = 2CD. Se P e o ponto de
AD tal que 6 ABP = 6 P AC, prove que 26 DP C = 6 BAC.
02. (IMO) Seja ABC um triangulo acut
angulo com circuncentro O. Seja P A uma altura do triangulo com
P no lado BC.
Considere que 6 BCA 6 ABC + 30 .
Prove que 6 CAB + 6 COP < 90 .
03. (IMO) Duas circunferencias 1 e 2 est
ao contidas no interior de uma circunferencia e sao tangentes
a em pontos distintos M e N , respectivamente. A circunferencia 1 passa pelo centro de 2 . A reta que
passa pelos dois pontos de interseccao de 1 e 2 intercepta em A e B. As retas M A e M B interceptam
1 respectivamente em C e D. Prove que CD e tangente a 2 .
04. (IMO) Considere um hex
agono convexo tal que para cada quaisquer dois
lados opostos verifica-se
a seguinte propriedade: a distancia entre os seus pontos medios e igual a 3/2 vezes a soma dos seus
comprimentos. Demonstre que todos os
angulos do hex
agono sao iguais.
(Um hex
agono convexo ABCDEF tem tres pares de lados opostos: AB e DE, BC e EF , CD e F A).
05. (Sao Petersburgo) Seja AL uma bissetriz interna do triangulo ABC, com L sobre BC. As retas paralelas
1 e 2 passam por B e C, respectivamente, e sao equidistantes de A. Os pontos M e N pertencem a 1 e
2 , respectivamente, e sao tais que os pontos medios de LM e LN pertencem a AB e AC, respectivamente.
Prove que LM = LN .
06. (R
ussia) No triangulo ABC, o ex-incrculo relativo a A toca o lado BC em A . Tracamos uma reta que
passa por A e e paralela `
a bissetriz interna de 6 BAC. Tracamos retas analogas para os outros dois lados.
Prove que essas tres retas sao concorrentes.
O resto do artigo sera para apresentarmos algumas ideias sinteticas para problemas de Geometria que
n
ao sao novas, mas que sao ao mesmo tempo simples e profundas. E o mais interessante: sao essencialmente
tecnicas que, se n
ao resolvem problemas completamente, podem orientar e estruturar as contas.

2.

Crculo de Apol
onio

S
ao dados dois pontos A e B e um real k > 0. Qual e o lugar geometrico dos pontos X do plano tais que
AX
= k?
BX
Sabemos a resposta para k = 1: a mediatriz de AB. E para k 6= 1?
Teorema 2.1.
plano tais que

Dados dois pontos distintos A e B e o real 0 < k 6= 1, o lugar geometrico dos pontos X do
AX
=k
BX

e um crculo cujo centro est


a sobre a reta AB; tal lugar geometrico e denominado crculo de Apolonio de A
e B e raz
ao k.
Demonstra
c
ao
A demonstracao e incrivelmente simples: geometria analtica. Podemos supor, sem perda de generalidade,
que A = (1; 0) e B = (0; 0). Se X = (x; y),
AX
= k AX 2 = k 2 BX 2
BX
(x 1)2 + y 2 = k 2 (x2 + y 2 )
1
2
x 2
=0
x2 + y 2 + 2
k 1
k 1
2
2


k
1
+ y2 =
,
x + 2
k 1
k2 1






ou seja, o lugar geometrico e o crculo de centro k211 ; 0 e raio k2k1 . Note que o centro pertence `a reta
AB, que admite equacao y = 0.
N
ao estamos interessados nessa demonstracao e sim, nas propriedades do crculo de Apolonio. O fato de
o lugar geometrico ser um crculo j
a e interessante per se: a partir de uma simples proporcao, conseguimos um
crculo e, conseq
uentemente,
angulos iguais; a conex
ao entre igualdade de raz
oes entre segmentos e igualdade
de angulos est
a estabelecida.
Mas ainda h
a muitas outras conex
oes.
No que se segue, e um crculo de Apolonio de A e B.
2.1.

Crculo de Apol
onio e bissetrizes

Crculos indicam
angulos iguais, mas quais? A resposta reside em outro teorema simples envolvendo raz
oes,
o teorema das bissetrizes:
Teorema 2.2. (Teorema das Bissetrizes). Se a bissetriz interna e a bissetriz externa de 6 ACB cortam AB
em T e U , respectivamente, ent
ao
AT
AU
AC
=
=
BT
BU
BC
A partir desse importante teorema, podemos provar o

Lema 2.1. Seja T a intersecao de e o segmento AB. Ent


ao e o lugar geometrico dos pontos P tais
que P T e bissetriz interna de 6 AP B.
Demonstra
c
ao

Imediato do teorema das bissetrizes, pois como P e T pertencem a ,


AT
AP
=
BT
BP

Reforcando: essa propriedade e o fato do lugar geometrico ser um crculo estabelece uma ligacao direta
entre raz
oes entre segmentos e
angulos, como veremos no proximo exemplo.
Exemplo 2.1.
(Polonia) Seja ABCD um quadril
atero c
oncavo, sendo o angulo interno 6 DAB maior que 180 e AB CD =
AD BC. Seja P o simetrico de A em relacao a BD. Prove que 6 P CB = 6 ACD.
Observacao: Esse problema foi proposto por um dos membros do forum Mathlinks (cadastre-se em
http://www.mathlinks.ro/) para mim via mensagem particular. Achei engracado porque eu nem conhecia
o cara e porque, de alguma forma, ele achou que eu ia resolver esse problema muito facilmente. Eu consegui
resolver, mas so depois de alguns dias pensei no crculo de Apolonio.
Eu ja sabia da definicao de crculo de Apolonio, mas nunca parei para pensar em suas propriedades.
O lema acima, por exemplo, veio durante a resolucao do problema. Foi o primeiro problema que vi cuja
resolucao envolvia o crculo de Apol
onio de modo t
ao crucial e que mostrou sua ubiq
uidade.
Resolu
c
ao
BA
BC
Usar crculo de Apol
onio aqui e razoavel se reescrevermos a condicao AB CD = AD BC como DA
= DC
.
Alem disso, sendo P o simetrico de A em relacao a BD, os triangulos BP D e BAD sao congruentes, logo
BA
BC
BP
rculo de Apolonio de B e D.
DP = DA = DC , ou seja, P , A e C pertencem a um mesmo c

Levando em conta ainda que queremos provar uma igualdade de angulos e que o crculo de Apolonio e
o lugar geometrico de vertices de bissetrizes, seja T o ponto de intersecao de e BD. Ent
ao CT e bissetriz

de 6 BCD e, conseq
uentemente, 6 BCT = 6 T CD.

a
A

Observe na figura que para chegarmos em 6 P CB = 6 ACD basta provar que 6 P CT =


isso e equivalente a AT = T P , o que decorre da simetria entre A e P em relacao a BD.
2.2.

T CA. Mas

Crculo de Apol
onio e qu
adruplas harm
onicas

A propria definicao do crculo de Apol


onio nos leva `a definicao de quadruplas harmonicas: sendo T e U as
intersecoes de um crculo de Apol
onio de A e B com o segmento que liga esses pontos. Ent
ao, de
AT
AU
=
BT
BU
conclumos que H(AB, T U ), ou seja, A, B, T e U formam uma quadrupla harmonica. Aqui, usamos a
notacao de [2].
Observe que a mediatriz de AB pode ser interpretada como uma circunferencia que passa pelo ponto
do infinito onde todas as retas paralelas a AB se cruzam. Isso corresponde ao caso em que um dos pontos
da quadrupla harm
onica e o ponto medio de AB.
Enquanto crculos de Apol
onio fazem aparecer quadruplas harmonicas, podemos pensar no processo
inverso, encontrando crculos de Apol
onios sem mesmo termos uma proporcao entre segmentos.
Como definir qu
adruplas harm
onicas sem contas? Use a definicao projetiva!
Lema 2.2. Seja ABC um triangulo e AM , BN e CP cevianas concorrentes. Se P N corta BC em Q,
ent
ao M Q e o diametro de um crculo de Apolonio de B e C.

Demonstra
c
ao
A demonstracao decorre da construcao do conjugado harmonico de [2], mas a repetimos aqui por completeza.

A
P

Do teorema de Ceva no triangulo ABC,


AN CM BP

=1
CN BM AP

()

Aplicando o teorema de Menelaus no triangulo ABC, reta N P Q,


AN CQ BP

=1
CN BQ AP
Comparando () e (), obtemos

CM
BM

CQ
BQ

()

e acabou.

Isso e u
til em problemas como o proximo exemplo.
Exemplo 2.2.
(Teste de Selecao, China) Sejam E e F as intersecoes dos lados opostos do quadril
atero convexo ABCD, cujas
diagonais cortam-se em P . Seja O sobre EF tal que OP e perpendicular a EF . Prove que 6 BOC = 6 AOD.
Resolu
c
ao

A
B

P
D
C

Voce consegue perceber a semelhanca entre as duas u


ltimas figuras? A diferenca aqui e que, em vez
de considerar a qu
adrupla ordenada H(EF, M Q), vamos considerar H(BD, P Q). Eles sao uma quadrupla
ordenada por causa das cevianas AP , DE e BF do triangulo ABD, que concorrem em C. Por estranho
que pareca, o teorema de Ceva ainda funciona para essas cevianas e, portanto, a demonstracao do u
ltimo
lema tambem.
Assim, P Q e diametro de um crculo de Apolonio de B e D. Como 6 P OQ e reto, O pertence a esse
crculo e, do lema sobre bissetrizes, OP e bissetriz de 6 BOD, ou seja, 6 BOP = 6 P OD.
Observando que 6 BOC = 6 BOP + 6 P OC e 6 AOD = 6 P OD + 6 AOP (se voce fez o seu proprio
desenho, pode ser que voce tenha que trocar ambos os sinais de + por ), so falta provar que 6 AOP = 6 P OC,
que e uma igualdade bem analoga `
a que provamos.
De fato, a demonstracao e analoga: agora considere o triangulo ACD e as cevianas (ainda mais
estranhas) AE, CF e DP , concorrentes em B. Obtemos, ent
ao, mais uma quadrupla, ordenada, H(AC, P M ).
O resultado, ent
ao, segue, j
a que O tambem pertence ao crculo de Apolonio de A e C de diametro P M .
Alguns exerccios para voce praticar.
Exerccios
07. Prove o seguinte
Lema 2.3. (Crculo de Apol
onio e inversao.) Prove que se , de centro O e raio r, e um crculo de Apolonio
de A e B, ent
ao, ao realizarmos uma inversao com centro O e raio r, A e o inverso de B.
08. (Teste de Selecao, Servia e Montenegro) Sejam M e N pontos distintos do plano do triangulo ABC
tais que AM : BM : CM = AN : BN : CN . Prove que M N contem o circuncrculo de ABC.
09. Seja I o incentro do triangulo ABC e D a intersecao de AI e BC. Seja M um ponto qualquer sobre o
circuncrculo de IBC. Prove que a reta M I bissecta o angulo 6 AM D.
10. (Teste de Selecao, EUA) Considere todos os triangulos n
ao is
osceles ABC tais que AC 2 +BC 2 = 2AB 2 .
As cevianas CM e CD sao respectivamente a mediana e a bissetriz interna relativas a C, com M e D sobre
AB. O ponto E e tal que D e o incentro de CEM . Prove que exatamente uma das raz
oes
CE
,
EM

EM
,
MC

MC
CE

e constante.
GA
Dica: prove que se G e o baricentro de ABC ent
ao GB
=
n
ao? Considere depois o simetrico de G em relacao a AB.

CA
CB .

A G pertence a um crculo especial,

11. (Torneio das Cidades) O


angulo 6 COD foi obtido da rotacao do angulo 6 AOB, de modo que OC
corresponde a OA e OD, a OB. Dois crculos tangenciam os angulos 6 AOB e 6 COD, respectivamente, e
se cortam em E e F . Prove que 6 AOE = 6 DOF .
Dica: Sejam O1 e O2 os centros dos crculos e r1 e r2 , os seus respectivos raios. Ent
ao
acontece que h
a segmentos que medem r1 e outros que medem r2 .

OO1
OO2

r1
r2 .

Mas

12. Dado o triangulo ABC, encontre o lugar geometrico dos pontos P interiores ao triangulo tais que
6

AP C 6 ABC = 6 AP B 6 ACB

Dica: o lugar geometrico comeca com crculo e termina com Apolonio. Use uma inversao com p
olo
em A.
Observacao: com esse resultado, o problema 2 da IMO 1996 e praticamente imediato: Seja P um ponto
interior ao triangulo ABC tal que 6 AP C 6 ABC = 6 AP B 6 ACB. Sejam D e E os incentros dos
triangulos AP B e AP C, respectivamente. Prove que as retas BD, CE e AP passam por um ponto comum.
De qualquer forma, a dificuldade desse problema e igual `a do anterior.

3.

Rota
co
es e Roto-Homotetias

Ao resolver o problema 5 da IMO 2005, eu notei que muitas pequenas coisas foram cruciais para a sua
resolucao (pelo menos, a que eu obtive):
(a) Uma ou mais figuras bem-feitas (no meu caso, tres figuras);
(b) Fazer um chute certeiro;
(c) Uma rotacao.
Depois, consegui generalizar o problema de modo que no item (c) trocamos rotacao por roto-homotetia.
Mas o que e uma rotacao? E uma roto-homotetia?
Defini
c
ao 3.1. Rotacao de uma figura F de um angulo em torno de um centro O no sentido anti-horario
(horario) e uma transformacao geometrica que associa a cada ponto P de F o ponto P tal que OP = OP
e 6 P OP = , sendo este
angulo orientado no sentido anti-horario (horario).

F
A

B
a
a
O

Antes de roto-homotetia, vamos definir homotetia.


Defini
c
ao 3.2. Homotetia de uma figura F com centro O e raz
ao k, sendo k um n
umero real positivo, e
uma transformacao geometrica que associa a cada ponto P de F o ponto P sobre a semi-reta OP , de origem
O, tal que OP = k OP .

O
B

B
Agora, definir roto-homotetia e simples:
Defini
c
ao 3.3. Roto-homotetia de uma figura F de um angulo em torno de um centro O no sentido
anti-horario (horario) e raz
ao k > 0 e uma transformacao geometrica que associa a cada ponto P de F o
ponto P tal que OP = k OP e 6 P OP = , sendo este angulo orientado no sentido anti-horario (horario).
Ou seja, e uma rotacao seguida de uma homotetia de mesmo centro.

a
A

O
a

A
OK, agora sabemos o que e roto-homotetia. Mas o que sao legais sao as suas

3.1.

Propriedades

Alem da semelhanca entre a figura e a imagem, roto-homotetias induzem mais duas semelhancas.
Lema 3.1. Se uma roto-homotetia (ou rotacao) de centro O leva A a A e B a B , ent
ao OAB e OA B

sao semelhantes (ou congruentes), assim como OAA e OBB .


Demonstra
c
ao

OB
OA
OA
Ambas sao decorrentes do caso LAL, pois OA
OB
= OB
e
OA = OB

6
6
angulo de rotacao) e, portanto, AOB = A OB (veja a figura anterior).
6

AOA =
6

BOB (iguais ao

Eu gosto de chamar essas semelhancas de semelhancas autom


aticas.
Exemplo 3.1.
(Rioplatense) No triangulo ABC, 6 BAC = 45 . P e Q sao pontos no interior de ABC tais que 6 CBP =
P BQ = 6 QBA = 6 ABC/3 e 6 BCP = 6 P CQ = 6 QCA = 6 ACB/3. Sejam D e E as projecoes ortogonais
de P sobre AB e AC, respectivamente. Prove que Q e o ortocentro do triangulo ADE.
6

Resolu
c
ao
Seja 6 ABC = 3. Logo 6 CBP = 6 P BQ =
BCP = 6 P CQ = 6 QCA = 45 .
6

QBA = ,
6

ACB = 180 45 3 = 135 3 e

45

Q
E

D
a

a
a

45o- a
45o- a
45o- a
C

Observe que, pelo caso AA, os triangulos DBP e QBC sao semelhantes. Ao encontrar uma semelhanca,
pode ser interessante pensar em alguma roto-homotetia que leve um triangulo ao outro. No caso, existe: a
BP
roto-homotetia de centro B, de no sentido anti-horario e raz
ao BC
leva QBC em DBP , ou seja, D = Q

e P = C . Temos, ent
ao, a semelhanca autom
atica entre os triangulos BQQ = BQD e BCC = BCP (a
justificativa e igual `
a do lema, e n
ao e nada extraordinario: note que, da semelhanca entre DBP e QBC,
BP
DB
6
6
QB = BC , e DBQ = P BC).

Mas, enfim, da semelhanca, 6 DQB = 6 P CB = 45 e, portanto, 6 QDA = 6 DQB + 6 DBQ =


45 + = 45 . Logo o
angulo entre as retas DQ e AE e 180 45 45 = 90 , ou seja, a reta DQ
contem uma altura do triangulo ADE. Analogamente, a reta QE contem outra altura do mesmo triangulo
e, conseq
uentemente, Q e o ortocentro de ADE.

Exerccios
13. (IMO) Seja ABCD um quadril
atero convexo e fixado com BC = DA e BC n
ao paralelo a DA. Sejam
E e F dois pontos variaveis sobre os interiores dos segmentos BC e DA, respectivamente, tais que BE = DF .
As retas AC e BD cortam-se em P ; as retas BD e EF cortam-se em Q; as retas EF e AC cortam-se em R.
Quando variamos E e F , obtemos diferentes triangulos P QR. Prove que os circuncrculos desses
triangulos tem um ponto comum diferente de P .
14. (Cone Sul) Sejam ABCD um quadrado (sentido hor
ario) e P um ponto qualquer pertencente ao interior
do segmento BC. Constr
oi-se o quadrado AP RS (sentido hor
ario).
Demonstrar que a reta CR e tangente `a circunferencia circunscrita ao triangulo ABC.
15. (Alemanha) Dos crculos 1 e 2 interceptam-se em A e B. Uma reta passa por B e corta 1 em C 6= B
e 2 em E 6= B. Outra reta passa por B e corta 1 em D 6= B e 2 em F 6= B. Suponha que B est
a entre
C e E e entre D e F . Sejam M e N os pontos medios de CE e DF , respectivamente.
Prove que os triangulos ACD, AEF e AM N sao semelhantes.
4.

Conjugados Isogonais em Rela


c
ao a um Tri
angulo

Antes de definir conjugados isogonais, vamos citar e provar um teorema importante e bastante u
til para
nossos prop
ositos.
4.1.

O Teorema de Ceva Trigonom


etrico

Teorema 4.1. Sejam AM , BN e CP cevianas do triangulo ABC. Ent


ao essas cevianas sao concorrentes
se, e somente se,
sen 6 CAM sen 6 ABN sen 6 BCP

=1
sen 6 M AB sen 6 N BC sen 6 P CA
A

a2

a1

T
g2

b1

g1

b2
B

Uma maneira simples de memorizar a relacao acima e marcar os angulos 1 , 2 , 1 , 2 , 1 , 2 como na


figura acima, em sentido hor
ario, e escrever
sen 1 sen 1 sen 1

=1
sen 2 sen 2 sen 2
Demonstra
c
ao
Para a demonstracao da ida, basta aplicar a lei dos senos nos triangulos AT B, BT C e CT A e multiplicar
as relacoes obtidas
TA
TB
TC
sen 1
sen 1
sen 1
,
,
=
=
=
TB
sen 2
TC
sen 2
TA
sen 2
A volta e um pouco mais trabalhosa, mas e analoga `a demonstracao (sintetica) do outro teorema de
Ceva. Suponha, por absurdo, que a relacao vale mas que as cevianas n
ao sao concorrentes. Trace, ent
ao,
uma ceviana CP que passa pela intersecao de AM e BN , e sejam = 6 BCP e = 1 + 2 , de modo que
6 P CA = e 2 = 1 .

a2

a1

P
P
N

T
b1

g2
q

b2
B

g1

Da ida que j
a demonstramos,
sen 1 sen 1
sen

=1
sen 2 sen 2 sen( )
Alem disso, por hip
otese

sen 1 sen 1 sen 1

=1
sen 2 sen 2 sen 2

Comparando as duas u
ltimas equacoes e substituindo 2 = 1 , obtemos
sen
sen 1
=
sen( )
sen( 1 )

Invertendo e aplicando o bom e velho truque da co-tangente:


sen cotg cos = sen cotg 1 cos cotg = cotg 1 = 1

Vale ressaltar que, sendo cotg uma funcao injetora em ]0; [, o teorema acima, a exemplo do outro
teorema de Ceva, vale quando T est
a fora do triangulo ABC tambem.
Estamos prontos para definir conjugados isogonais.
Defini
c
ao 4.1. Dado um triangulo ABC, o conjugado isogonal em relacao a ABC de um ponto T do
plano de ABC e obtido refletindo as retas T A, T B e T C em relacao `as bissetrizes internas de ABC que
passam por A, B e C, respectivamente. As retas resultantes sao concorrentes no isogonal T 1 de T .
A seguir, as linhas pontilhadas sao as bissetrizes, e as cevianas cinzas sao as reflex
oes das cevianas
pretas.
A

T -1

T
B

Por que as cevianas cinzas sao concorrentes? Isso decorre de duas aplicacoes do teorema de Ceva
trigonometrico: primeiro com as cevianas concorrentes em T e depois, com as cevianas concorrentes em
T 1 , que formam os mesmos
angulos que as outras cevianas, porem no sentido contrario.
Na verdade, pode ocorrer de as tres cevianas serem paralelas. Isso ocorre se, e somente se, T est
a sobre
o circuncrculo de ABC; nesse caso, pensamos projetivamente, ou seja, o conjugado isogonal e um ponto do
infinito.
Alias, dois pontos bastantes conhecidos sao conjugados isogonais: o circuncentro e o ortocentro. Tente
provar isso.
4.2.

Para que servem isogonais?

O que e mais u
til em conjugados isogonais e simplesmente que as cevianas sao reflex
oes umas das outras em
relacao `as bissetrizes, e isso costumam levar a algumas igualdades entre angulos um pouco mais difceis de
obter ou mesmo de se imaginar com contas.
Exemplo 4.1.
ao
Voltemos `a situacao do exemplo de roto-homotetia, da Rioplatense. Suponha agora que o angulo 6 BAC n
esperar muito que o ponto Q seja ortocentro de ADE e, de fato, nem sempre
meca necessariamente 45 . E
e. Mas a reta AQ ainda e perpendicular a DE. Prove esse fato.

Resolu
c
ao
Seja = 6 P AD. Ent
ao 6 AP D = 90 e, como
inscritvel. Logo 6 AED = 6 AP D = 90 .
6

ADP e
6

AEP sao retos, o quadril


atero ADP E e

q
q
Q

90 - q

90 - q
a

b
b
b

Olhando a figura, note que basta provarmos que 6 QAC = . A e que entram os conjugados isogonais.
Como 6 P BC = 6 QBA e 6 BCP = 6 QCA, os pares de retas BP ; BQ e CP ; CQ sao simetricos entre si
em relacao `as bissetrizes de 6 ABC e 6 ACB, respectivamente. Ou seja, P e Q sao conjugados isogonais
e, portanto, 6 P AB e 6 QAC tambem sao iguais. Logo 6 QAC = e o angulo entre as retas AQ e DE e
180 (90 ) = 90 .
Note que para provar o resultado na conta, bastaria repetir a demonstracao da volta do teorema de
Ceva trigonometrico. Mas o que e mais interessante e que, sabendo da existencia dos conjugados isogonais,
e natural pensar nessa solucao. Em contraste, fazer a conta sem pensar em conjugados isogonais n
ao parece
ser t
ao natural assim. Ent
ao d
a para pensar que os conjugados isogonais nos economizou n
ao so fazer a
conta, mas mostrou onde fazer as contas relevantes.
O proximo exemplo mostra o verdadeiro poder dos conjugados isogonais.

Exemplo 4.2.
(IMO) Num quadril
atero convexo ABCD a diagonal BD n
ao e bissetriz do angulo 6 ABC nem do angulo
CDA. Um ponto P no interior de ABCD satisfaz
6

P BC = 6 DBA

e
6

P DC = 6 BDA.

Prove que os vertices do quadril


atero ABCD pertencem a uma mesma circunferencia se, e somente se,
AP = CP .

Resolu
c
ao
Na figura a seguir, prolongamos AP de modo que encontre BD em Q.

P
b

a
B
b

q
q

C
Note que, de acordo com as marcacoes da figura, 6 BP Q = 6 CP D. Isso ocorre porque, sendo 6 P BC =
DBA e 6 P DC = 6 BDA, os pares de retas BA; BC e DA; DC sao simetricos em relacao `as bissetrizes de
6 P BD e 6 P DB, respectivamente. Assim, os pontos A e C s
ao conjugados isogonais em relacao ao triangulo
BP D e, portanto, as retas P A e P C sao simetricas em relacao `a bissetriz de 6 BP D e realmente temos
6 BP Q = 6 CP D.
6

Caso voce queira identificar as ternas de cevianas respectivamente isogonais, elas sao P C, BC, DC e
P A, BA, DA.
Agora, algumas contas terminam o problema: aplicando a lei dos senos aos triangulos AP B e CP D,
temos
AP
AB
CP
CD
=
e
=
sen
sen
sen
sen
Dividindo membro a membro e denotando RXY Z o circunraio do triangulo XY Z, obtemos
AB
AP
AP sen
=

=
CP sen
CD
CP

6
6

AB
sen
CD
sen

RABD
AP
=
CP
RBCD

()

Alem disso, supondo sem perda de generalidade que P est


a no interior do triangulo ABD, temos
ABD > 6 CBD e 6 ADB > 6 CDB. Portanto 180 6 ABD 6 ADB < 180 6 CBD 6 CDB
BAD < 6 BCD.

Note que tudo o que fizemos ate agora n


ao depende de ABCD ser inscritvel ou de P A = P C, ou seja,
n
ao assumimos nenhuma das hip
oteses da afirmacao que queremos provar.
Somente agora vamos provar a afirmacao do enunciado: de ()
P A = P C RABD = RBCD
Isto e, os circuncrculos dos triangulos ABD e BCD, de lado comum BD, tem o mesmo raio. Isso quer
dizer que ou os circuncrculos coincidem ou sao simetricos em relacao a BD. Mas essa u
ltima possibilidade
a vimos que n
ao e possvel.
implica 6 BAD = 6 BCD, que j

Exerccios
16. Prove que o ortocentro e o circuncentro de um triangulo sao conjugados isogonais em relacao ao mesmo
triangulo.
17. Sejam T e T 1 pontos conjugados isogonais em relacao ao triangulo ABC. Prove que as seis projecoes
ortogonais de T e T 1 sobre os lados (ou prolongamentos) de ABC pertencem a um crculo com centro no
ponto medio de T T 1 .
Note que esse u
ltimo problema generaliza o crculo dos nove pontos.
18. (Ira) Os pontos M e M sao conjugados isogonais no triangulo ABC. Sejam P , Q e R as projecoes
ortogonais de M sobre as retas BC, AC e BC, respectivamente. Defina P , Q , R analogamente para M .
As retas QR e Q R cortam-se em D; RP e R P cortam-se em E; e P Q e P Q cortam-se em F . Prove que
as retas AD, BE e CF sao paralelas.
Dica: use o exerccio anterior.
5.

Refer
encias Bibliogr
aficas

[1] Carlos Shine, Geometria com Contas, Revista Eureka! 17. Uma boa introducao para quem quer comecar
a fazer problemas de geometria com trigonometria e geometria analtica. L
a tem bastantes problemas,
incluindo alguns da IMO e alguns exemplos desse artigo.
[2] Luciano Castro, Introducao `
a Geometria Projetiva, Revista Eureka! 8. Tudo o que voce precisa saber
sobre geometria projetiva e resolver problemas de olimpadas.
[3] Forum Mathlinks, http://www.mathlinks.ro/. Tirei muitos problemas de la, e algumas solucoes
tambem. Em particular, tem um curso bem detalhado com varias propriedades de conjugados isogonais
em
http://www.mathlinks.ro/Forum/viewtopic.php?t=18472.
[4] Mais informacoes sobre crculo de Apolonio e conjugados isogonais podem ser encontradas no Mathworld,
da Wolfram, mais especificamente em
http://mathworld.wolfram.com/ApolloniusCircle.html
e
http://mathworld.wolfram.com/IsogonalConjugate.html
Por fim, gostaria de agradecer ao professor Edmilson Motta por revisar o artigo e fazer sugestoes
construtivas, alem de todo o apoio.

Geometria com Contas


` vezes precisamos de mais elementos para resolver problemas de geometria. Pode-se tracar novos elementos
As
na figura que possam ajudar ou fazer algumas contas. Mostraremos algumas tecnicas para fazer algumas
contas que ajudam (e ate resolvem!).
Em geral, pode-se pensar em problemas de geometria seguindo esses passos:
(i) Faca a figura do problema (praticamente nenhum problema vem com figura), bem grande e com certa
precisao (ou seja, use a regua e o compasso, mas nao e necessario muito rigor).
(ii) Mexa um pouco com os elementos da figura. Algo que e sempre u
til e fixar um certo n
umero de angulos
(de preferencia, o menor n
umero possvel, de modo que os angulos marcados determinem a figura a nao
ser, e claro, que acrescentar algum outro angulo adicione alguma simetria algebrica u
til) e calcular todos
os outros angulos possveis (se os angulos que voce escolheu determinam a figura, e possvel calcular
todos os outros, de um jeito ou de outro). Procure quadrilateros inscritveis para ajudar. Se necessario,
faca conjecturas (e para isso que voce fez um desenho bem feito!). Alguns problemas de geometria ja
sao resolvidos nesse passo!
(iii) Se o problema ainda nao foi resolvido, e hora de elaborar uma estrategia para resolver o problema, ou
seja, determinar quais calculos devem ser feitos. Nada de fazer calculos sem planeja-los!
(iv) Execute sua estrategia. Lembre-se sempre de ter uma meta em mente (algo do tipo precisamos calcular
tal angulo) e, se voce estiver numa prova, de controlar seu tempo e o tamanho da conta (nao deixe a
conta crescer muito; a falta de controle e um fermento muito poderoso para contas).
claro que esses passos nao sao precisos e que, para domina-los, e preciso muito treino e, por que nao,
E
aprender algumas tecnicas.
1.

Trigonometria

Muitos problemas de geometria podem ser resolvidos com o auxlio da trigonometria. As formulas que voce
deve saber sao basicamente essas quatro:
sen(a + b) = sen a cos b + sen b cos a
sen(a b) = sen a cos b sen b cos a
cos(a + b) = cos a cos b sen a sen b
cos(a b) = cos a cos b + sen a sen b
A partir dessas voce pode deduzir essas outras, que na verdade sao as mais u
teis para nos e que tornam
a trigonometria tao poderosa.
Transformando somas em produtos
Transformando produtos em somas





1
x+y
xy
sen a sen b = cos(a b) cos(a + b)
sen x + sen y = 2 sen
cos
2
2
2





1
x

y
x
+
y
cos a cos b = cos(a b) + cos(a + b)
sen x sen y = 2 sen
cos
2
2
2

1




sen a cos b = sen(a b) + sen(a + b)
x
+
y
x

y
2
cos x + cos y = 2 cos
cos
2
2




x+y
xy
cos x cos y = 2 sen
sen
2
2
Por fim, relembramos a lei dos senos e a lei dos co-senos. No triangulo ABC, seja AB = c, AC = b,

BC = a, 6 A = , 6 B = e 6 C = . O circunraio de ABC e R.
a
b
c
=
=
= 2R
sen
sen
sen
a2 = b2 + c2 2bc cos
b2 = a2 + c2 2ac cos
c2 = a2 + b2 2ab cos
A lei dos senos, por envolver proporcoes (que sao mais simples) e elementos adicionais do triangulo (o
circunraio), e particularmente u
til.
Vamos resolver alguns problemas e mostrar algumas tecnicas de calculo.
1.1.

Conven
c
ao

Sempre que houver um triangulo ABC, , e sao as medidas dos angulos


respectivamente.
1.2.

BAC,
6

ABC e
6

ACB,

Um come
co e o truque da co-tangente

Exemplo 1.1.
(Prova de Selecao para a IMO) Seja uma circunferencia de centro O tangente aos lados AB e AC do
triangulo ABC nos pontos E e F . A reta perpendicular ao lado BC por O intercepta EF no ponto D.
Mostre que A, D e M (ponto medio de BC) sao colineares.
Resolu
c
ao
Primeiro, um bom desenho, com todos os angulos que pudermos marcar (a tecnica do arrastao e bastante u
til
e por isso que voce deve fazer um desenho grande!!). Note que os angulos do triangulo ABC ja determinam
toda a figura (para perceber isso, note que se construir ABC todos os outros elementos da figura ja estao
determinados).

p
2
p
2

a
2

D
b

E
a
2

a
2

a
2

O
G

b
B

g
M

sempre bom justificar os calculos. Seja P a intersecao e BC e da reta perpendicular a BC. Como
E
6 BEO e 6 BP O s
ao retos, o quadrilatero BP OE e inscritvel, de modo que 6 DOE = 6 EBM = . Analogamente, 6 DOF = .
A reta AO e bissetriz e AOEF e inscritvel, logo 6 OEF = 6 OF E = /2.
Mas, como provar que A, D e M estao alinhados? Uma maneira e provar que
exemplo. Para isso, e so calcular os dois angulos.
6

BAD = 6 BAM , por

Como calcularemos = 6 BAD? Veja o triangulo ADE. Sendo r o raio de , com uma lei dos senos
calculamos DE. AE pode ser facilmente calculado. Como ja conhecemos 6 AED (viu como e bom fazer o
arrastao?), temos elementos suficientes para calcular .
Para calcular = 6 BAM , usaremos o triangulo BAM , da qual conhecemos BM , AB e 6 ABM .
Ja temos uma estrategia. Vamos executar o plano!

A
f

p
2

a
D
2
a b
2

E
r

No triangulo ODE,
r
DE
r sen
=
DE =
sen
sen( + 2 )
sen( + 2 )
(note que 6 ODE = ( + /2) utilizamos o fato de que sen x = sen( x) para todo x real; utilizaremos
bastante esse fato e o fato sen(/2 x) = cos x)
Sendo o triangulo AEO retangulo em E, obtemos AE = r cotg(/2).
No triangulo ADE,
DE
AE
=
sen
cos( + 2 )

()

Quando temos uma equacao do tipo


a
b
=
,
sen x
sen(x + )
e queremos determinar x, utilizamos o truque da co-tangente:
a
b
sen(x + )
b
sen x cos + sen cos x
b
b
=

=
cos + sen cotg x =
sen x
sen(x + )
sen x
a
sen x
a
a
e podemos isolar cotg x.

Voltemos a (). Substituindo DE e AE e utilizando o truque da co-tangente, temos


cos


2

cotg + sen


2

cotg( 2 ) sen( + 2 )
cos( 2 ) sen( + 2 ) sen2 ( 2 ) sen
cotg =
sen
sen( 2 ) sen cos( 2 )
2 sen( + 2 ) cos( 2 ) 2 sen2 ( 2 ) sen
sen sen
sen( + ) + sen 2 sen2 ( 2 ) sen
cotg =
sen sen
sen( + ) + sen (1 2 sen2 ( 2 ))
cotg =
sen sen
sen( + ) + sen cos
cotg =
sen sen
cotg =

Calculemos . Uma pratica normal em trigonometria e adotar o circunraio de algum triangulo igual a
1/2, de modo que, pela lei dos senos, seus lados sejam iguais aos senos dos seus respectivos angulos opostos.
claro que so podemos fazer isso uma
Podemos fazer isso porque estamos so fixando o tamanho da figura. E
vez so em cada problema.
A

sen g

b
B

sen a
2

Nesse caso, facamos isso com 4ABC. Temos BM = BC/2 =


triangulo ABM ,

M
1
2

sen e AB = sen = sen( + ). No

AB
2 sen( + )
2 sen( + ) sen cos
BM
=
sen cotg + cos =
cotg =
sen
sen( + )
sen
sen sen
Puxa, os resultados de cotg e cotg sao diferentes! Na verdade, nao sao. Nunca perca a fe!
cotg = cotg sen( + ) + sen cos = 2 sen( + ) sen cos
sen( + ) = sen cos + sen cos ,

que e sempre verdade.

1.3.

Algumas identidades

Suponha que o circunraio do triangulo ABC e R = 1/2. Entao, c = AB = sen , b = AC = sen e


a = BC = sen . Alem disso, por exemplo,

O permetro do triangulo e 2p = 4 cos( 2 ) cos( 2 ) cos( 2 );

A area do triangulo e S = sen sen sen /2;

O inraio do triangulo e r = 2 sen( 2 ) sen( 2 ) sen( 2 );

cos + cos + cos = 1 + r/R;

p a = 2 cos( 2 ) sen( 2 ) sen( 2 ).

01.

Prove todas as identidades acima.

Exemplo 1.2.

(IMO) Sejam AH1 , BH2 e CH3 as alturas de um triangulo acutangulo ABC. A circunferencia inscrita
no triangulo ABC e tangente aos lados BC, CA, AB em T1 , T2 e T3 , respectivamente. Considere a reta
simetrica da reta H1 H2 relativamente a` reta T1 T2 , a reta simetrica da reta H2 H3 relativamente a` reta T2 T3 ,
a reta simetrica da reta H1 H3 relativamente a` reta T1 T3 . Prove que estas retas simetricas determinam um
triangulo cujos vertices pertencem a` circunferencia inscrita no triangulo ABC.

Resolu
c
ao

Esse e o u
nico problema 6 de IMO de geometria euclidiana nos u
ltimos 10 anos.

Primeiro, uma boa, e bem grande, figura. Vamos so desenhar a reta simetrica relacionada a T 2 T3 . H e

o ortocentro de ABC.

A
p -b
2
p g
2

g
H3
p a g
- 2 2
p a g
- P
2 2

H2

p -b
2

p a
2 2

T2

T3

l1
p - a -g = b

X3

b
B

g
H1

Facamos o arrastao: veja que AH2 HH3 e inscritvel, logo 6 AH3 H2 = . Seja P a intersecao de T2 T3 e
H2 H3 (so nao podemos escolher duas retas Ti Tj e Hi Hj concorrentes quando o triangulo ABC e equilatero;
tal caso e trivial). Como AT2 = AT3 , os angulos 6 AT2 T3 e 6 AT3 T2 medem ambos /2 /2. Assim,
6 H3 P T3 = 6 AT3 T2 6 P H3 T3 = /2 e, sendo `1 a reta sim
etrica da reta H2 H3 relativamente
a` reta T2 T3 , o angulo entre `1 e T2 T3 e igual tambem a /2 . Logo o angulo entre `1 e AB e
2(/2 /2 ) + = = , ou seja, `1 e BC sao paralelos. Definindo analogamente `2 e `3 , temos
`2 // AC e `3 // AB.

Com isso, ja sabemos que o triangulo determinado por `1 , `2 e `3 e semelhante a ABC, e com lados

homologos paralelos. Temos, entao, dois candidatos a tal triangulo:

C B

Estudando um caso particular (o triangulo equilatero, por exemplo), vemos que o candidato mais indicado e o da direita. Podemos, entao, calcular a distancia entre lados homologos nessa situacao e compararmos
com a distancia entre BC e `1 .
Assuma que o circunraio de ABC e 1/2, para termos BC = sen , CA = sen e BC = sen .
Vamos calcular a distancia entre BC e `1 . Seja X3 a intersecao de `1 e AB. A distancia de A a `1 e
AX3 sen . E a distancia desejada e AH1 AX3 sen . Bom, AH1 e facil de calcular: AH1 = AB sen =
sen sen . E AX3 ? AH3 e facil de calcular, AT3 tambem. Podemos calcular H3 T3 = AT3 AH3 e usar a
lei dos senos no triangulo P H3 X3 , com a ceviana P T3 . Maos a` obra!!
Para comecar, AH3 = AC cos = sen cos e AT3 = p sen , sendo p o semipermetro de ABC.
Portanto H3 T3 = p sen sen cos .
Pela lei dos senos no triangulo P H3 T3 ,
H3 T 3
P T3
=
sen
sen 6 H3 P T3
No triangulo P T3 X3 ,

X3 T3
P T3
=
sen
sen 6 X3 P T3

Dividindo as duas u
ltimas equacoes e tendo em vista que 6 H3 P T3 = 6 X3 P T3 , obtemos
X3 T3 =

p sen sen sen sen sen cos


sen
H3 T 3 =
sen
sen

Da lei dos co-senos (ela tambem e u


til de vez em quando!),
sen sen cos =
Logo, substituindo p =

sen2 + sen2 sen2


2

sen +sen +sen


,
2

sen sen + sen sen + sen2 sen2 sen2 + sen2


2 sen
sen sen + sen sen sen2 + sen2
=
2 sen

X3 T3 =

Enfim, podemos calcular AX3 = AT3 + X3 T3 . Veja que AT3 = p sen =

sen +sen +sen


.
2

sen ( sen + sen + sen ) sen sen + sen sen sen2 + sen2
2 sen
sen sen + 2 sen sen sen sen + sen2
=
2 sen

AX3 =

Enfim, a distancia entre `1 e BC e


sen sen + 2 sen sen sen sen + sen2
2
sen ( sen + sen + sen )
=
2

AH1 AX3 sen = sen sen

Na secao de identidades, voce deve provar que


 
 

p a = 2 cos
sen
sen
2
2
2


Logo a distancia entre `1 e BC e (ufa!)


d = 2 cos


2

sen

 
 

sen
sen
2
2

Agora calculemos a distancia entre os lados homologos dos triangulos ABC e o de lados respectivamente
paralelos aos lados de ABC.

A
a

C
a
r
I
r
a
A

Seja I o incentro do triangulo ABC. A distancia de I a BC e igual ao inraio r e a distancia de I a B 0 C 0


e r cos . Assim, a distancia entre BC e B 0 C 0 e

d0 = r + r cos = r(1 + cos ) = 2r cos2
2
Voce tem outra identidade para provar:
r = 2 sen


2

sen

 
 

sen
2
2

Logo
 

 

d = 2 2 sen
sen
sen
cos2
2
2
2
2
 






= 2 2 sen
cos
sen
sen
cos
2
2
2
2
2
 




= 2 cos
sen
sen
sen = d
2
2
2


Conseq
uentemente, `1 contem B 0 C 0 . Analogamente (ou voce acha que eu faria todas as contas de novo?),
`2 contem A0 C 0 e `3 contem A0 B 0 .
` vezes tracar novos elementos na figura tambem ajuda.
As
Exemplo 1.3.
(IMO) Seja P um ponto interior ao triangulo ABC tal que
6

AP C 6 ABC = 6 AP B 6 ACB

Sejam D e E os incentros dos triangulos AP B e AP C, respectivamente. Prove que as retas BD, CE e


AP passam por um ponto comum.
Resolu
c
ao
Seja = 6 AP C 6 ABC = 6 AP B 6 ACB.

a1

a2

g+q

b+q
P

Veja que podemos separar de e . Note que se ficar para baixo obtemos um quadrilatero

inscritvel, entao faremos isso.

a1

a2

g
G

a1 + g

a2 + b
q

q
P

O quadrilatero AF P G e inscritvel, logo 6 AF G = , ou seja, F G // BC.


O problema pede, na verdade, para provarmos que as bissetrizes de 6 ACP e 6 ABP se encontram sobre
AP . Sejam Q e R as intersecoes de BD e CE com AP . Devemos ter Q = R. Do teorema das bissetrizes,
AB
AQ
=
QP
BP
Como
Q = R AQ = AR

AR
AC
=
RP
CP

AQ
AR
AQ
AR
=

=
,
AP AQ
AP AR
QP
RP

e suficiente demonstrarmos que


AC
AB
=
BP
CP
Vamos, entao, calcular BP e CP . Sendo F G paralela a BC, temos F B = k AB e GC = k AC.
Aplicando a lei dos senos ao triangulo BF P , temos
BP
FB
k AB sen(1 + )
AB
sen
=
BP =

=
sen(1 + )
sen
sen
BP
k sen(1 + )
Analogamente,

AC
sen
=
CP
k sen(2 + )

Como 1 + e 2 + somam , o resultado esta demonstrado.

2.

Geometria Analtica

Quando aparecem problemas com muitos angulos retos e que envolvam so retas, geometria analtica a`s vezes
e indicada.
Exemplo 2.1.
(IMO) No quadrilatero convexo ABCD, as diagonais AC e BD sao perpendiculares e os lados opostos AB
e CD nao sao paralelos. Sabemos que o ponto P , onde se intersectam as mediatrizes de AB e CD, esta no
interior de ABCD.
Prove que ABCD e um quadrilatero cclico se, e somente se, os triangulos ABP e CDP tem areas
iguais.
Resolu
c
ao
Esse problema e perfeito para se resolver com geometria analtica: e muito facil colocar as coisas nos eixos
(tome como eixos as diagonais); tudo e muito facil de calcular analiticamente (mediatrizes e areas); e, por
fim, a u
nica condicao que poderia complicar, que e saber quando ABCD e cclico, pode ser facilmente
transformada na potencia da intersecao das diagonais em relacao ao seu circuncrculo.

(0;b) B

(c;0)

(a;0)

(0;d) D
Sejam, entao, A = (a; 0), B = (0; b), C = (c; 0) e D = (0; d). O quadrilatero ABCD e inscritvel se, e
somente se, OA OC = OB OD ac = bd. Facil, nao?
Seja P = (x; y). Como P pertence a`s mediatrizes de AB e CD, temos P A = P B e P C = P D.
P A = P B (x a)2 + (y 0)2 = (x 0)2 + (y b)2 2ax a2 = 2by b2
Analogamente, P C = P D 2cx c2 = 2dy b2 . Resolvendo o sistema obtido, temos


x = (a2 b2 )d(c2 d2 )b
2ax 2by = a2 b2

2(adbc)

2cx 2dy = c2 d2
(a2 b2 )c(c2 d2 )a
y=
2(adbc)

Tudo bem com os denominadores pois, como AB e CD nao sao paralelos, OA/OB 6= OC/OD
a/b 6= c/d ad bc 6= 0 (nunca se esqueca de verificar quando os denominadores sao nulos; essa
verificacao a`s vezes faz voce perceber que tem que estudar alguns casos em separado).
A area do triangulo P AB e igual a |D|/2, em que


x y 1


D = a 0 1 = ay bx + ab
0 b 1

Da mesma forma, a area do triangulo P CD e igual a |D 0 |/2, em que




x y 1


D0 = c 0 1 = cy dx + cd
0 d 1

Assim, devemos ter

| ay bx + ab| = | cy dx + cd|
Seria muito bom nos livrarmos do modulo. O sinal de D depende da ordem em que colocamos as
coordenadas no determinante. Se os pontos correspondentes estao dispostos no sentido anti-horario, D e
positivo; se estao no sentido horario, e negativo. Como P pertence ao interior de ABCD, P AB e P CD
tem a mesma orientacao, de modo que realmente podemos nos livrar do modulo. Logo, tirando o modulo e
substituindo x e y, temos que as areas de P AB e P CD sao iguais se, e somente se,
(a c)

(a2 b2 )d (c2 d2 )b
(a2 b2 )c (c2 d2 )a
+ (b d)
= ab cd
2(ad bc)
2(ad bc)

()

Nada de abrir tudo com pressa! Queremos ac = bd, e isso significa que provavelmente em algum momento
fatoraremos a equacao com ac bd como um dos fatores.
() (a2 b2 )(ac + bd c2 d2 ) + (c2 d2 )(ac + bd a2 b2 ) = 2(ab cd)(ad bc)
(ac + bd)(a2 + c2 b2 d2 ) a2 c2 a2 d2 + b2 c2 + b2 d2 a2 c2 b2 c2 + a2 d2 + b2 d2
= 2(a2 bd ab2 c acd2 + bc2 d)
ac(a2 + c2 ) bd(b2 + d2 ) acb2 acd2 + bda2 + bdc2 + 2(a2 c2 b2 d2 )
= 2(a2 bd ab2 c acd2 + bc2 d)
ac(a2 + c2 ) bd(b2 + d2 ) (acb2 acd2 + bda2 + bdc2 ) + 2(a2 c2 b2 d2 ) = 0
ac(a2 + c2 ) bd(b2 + d2 ) + ac(b2 + d2 ) bd(a2 + c2 ) + 2(ac bd)(ac + bd) = 0
(ac bd)(a2 + b2 + c2 + d2 ) (ac bd)(2ac + 2bd) = 0
(ac bd)((a c)2 + (b d)2 ) = 0
ac = bd ou (a = c e b = d)
Nao e possvel termos a = c e b = d pois ja vimos que ad 6= bc. Logo as areas de P AB e P CD sao iguais
se, e somente se, ac = bd.
A geometria analtica tem uma pequena desvantagem: nao passa de aplicacoes extensivas do teorema
de Pitagoras. Apesar de Pitagoras resolver problemas como o que acabamos de ver, mesclar um pouco as
contas com trigonometria e n
umeros complexos pode vir a calhar.
Agora, alguns problemas para voce pensar.
02. Seja ABC um triangulo acutangulo, M o ponto medio do segmento BC, P o ponto sobre o segmento
AM tal que P M = BM , H o pe da perpendicular de P a BC, Q o ponto de intersecao entre o segmento

AB e a reta que passa atraves de H e e perpendicular a P B e, finalmente, R o ponto de intersecao entre


o segmento AC e a reta que passa atraves de H e e perpendicular a P C. Mostre que o circuncrculo do
triangulo QHR e tangente a BC no ponto H.
03. No triangulo ABC, AB = AC. D e um ponto sobre o lado BC tal que BD = 2CD. Se P e o ponto de
AD tal que 6 ABP = 6 P AC, prove que 26 DP C = 6 BAC.
04. Um quadrilatero convexo esta inscrito em uma circunferencia de raio unitario. Demonstre que a
diferenca entre seu permetro e a soma das diagonais e maior do que zero e menor do que 2.
05. (IMO) O prolongamento da bissetriz AL do triangulo acutangulo ABC intercepta a circunferencia
circunscrita no ponto N . A partir do ponto L tracam-se perpendiculares LK e LM aos lados AB e AC,
respectivamente. Prove que a area do triangulo ABC e igual a area do quadrilatero AKN M .
06. (Ibero) A circunferencia inscrita no triangulo ABC e tangente aos lados BC, CA e AB nos pontos D,
E e F , respectivamente. AD corta a circunferencia num segundo ponto Q. Demonstrar que a reta EQ passa
pelo ponto medio de AF se, e somente se, AC = BC.
07. (IMO) Seja I o incentro do triangulo ABC. A circunferencia inscrita no triangulo ABC e tangente aos
lados BC, CA e AB nos pontos K, L e M , respectivamente. A reta que passa por B, paralela ao segmento
M K, intercepta as retas LM e LK nos pontos R e S, respectivamente. Prove que o angulo 6 RIS e agudo.
08. (Vietna) Seja ABC um triangulo e A0 , B 0 , C 0 pontos medios dos arcos BC, AC e AB do circuncrculo
de ABC, respectivamente. As retas A0 B 0 e A0 C 0 interceptam o lado BC em M e N , respectivamente.
Defina os pares de pontos P , Q e R, S analogamente. Prove que M N = P Q = RS se, e somente se, ABC e
equilatero.
09. (IMO) Seja ABC um triangulo acutangulo com circuncentro O. Seja P A uma altura do triangulo com
P no lado BC.
Considere que 6 BCA 6 ABC + 30 .
Prove que 6 CAB + 6 COP < 90 .
10. (IMO) Num triangulo ABC, seja AP a bissetriz de
de 6 ABC com Q no lado CA.
6

BAC com P no lado BC, e seja BQ a bissetriz

Sabemos que 6 BAC = 60 e que AB + BP = AQ + QB.


Quais sao os possveis valores dos angulos do triangulo ABC?
11. (Coreia) Sejam R e r o circunraio e o inraio, respectivamente, do triangulo ABC, e R 0 e r0 o circunraio
e o inraio, respectivamente, do triangulo A0 B 0 C 0 . Prove que se 6 C = 6 C 0 e Rr0 = R0 r entao os triangulos
sao semelhantes.
12. (Turquia) Sejam AC e PC a area e o permetro, respectivamente, do quadrilatero cclico C. Se a area e
o permetro do quadrilatero cujos lados sao tangentes ao circuncrculo de C sao A T e PT , respectivamente,
prove que

2
AC
PC

AT
PT
13. (EUA) Seja ABCD um trapezio isosceles com AB // CD. O incrculo do triangulo BCD toca CD em
E. Seja F um ponto da bissetriz de 6 DAC tal que EF CD. O circuncrculo do triangulo ACF corta a
reta CD em C e G. Mostre que o triangulo AF G e isosceles.
14. (Balcanica, adaptado) Seja ABC um triangulo acutangulo e M , N e P as projecoes ortogonais do
baricentro de ABC sobre seus lados. Prove que
2
[M N P ]
1
<

9
[ABC]
4
([XY Z] e a area do triangulo XY Z)

15. (Ibero) Dados dois crculos 1 e 2 , dizemos que 1 bissecta 2 quando se intersectam e a corda comum
e um diametro de 2 . Se 1 e 2 sao identicas, dizemos que 1 e 2 bissectam-se mutuamente. Considere
dois crculos fixos e nao concentricos 1 e 2 .
(a) Mostre que ha infinitos crculos que bissectam tanto 1 como 2 .
(b) Encontre o lugar geometrico do centro de .
16. (Ibero) Seja ABC um triangulo acutangulo com circuncrculo centrado em O. Seja AD, BE e CF
as alturas de ABC. A reta EF corta em P e Q.
(a) Prove que AO P Q.
(b) Se M e o ponto medio de BC, prove que
AP 2 = 2AD OM
17. (Sao Petersburgo) Seja AL uma bissetriz interna do triangulo ABC, com L sobre BC. As retas paralelas
`1 e `2 passam por B e C, respectivamente, e sao equidistantes de A. Os pontos M e N pertencem a ` 1 e
`2 , respectivamente, e sao tais que os pontos medios de LM e LN pertencem a AB e AC, respectivamente.
Prove que LM = LN .

Homotetias, composi
c
ao de homotetias
e o problema 6 da IMO 2008
Antes de comecar a discuss
ao, vamos enunciar o problema 6 da IMO 2008, que e a motivacao principal desse
artigo.
Problema 6, IMO 2008. Seja ABCD um quadril
atero convexo cujos lados BA e BC tem comprimentos diferentes. Sejam 1 e 2 as circunferencias inscritas nos triangulos ABC e ADC, respectivamente.
Suponhamos que existe uma circunferencia tangente `a reta BA de forma que A est
a entre B e o ponto
de tangencia, tangente `
a reta BC de forma que C est
a entre B e o ponto de tangencia, e que tambem seja
tangente `as retas AD e CD. Prove que as tangentes comuns exteriores a 1 e 2 se intersectam sobre .
claro que um problema de geometria n
razoavelmente difcil
E
ao pode ficar sem um bom desenho. E
desenhar a figura do problema e sugerimos que o leitor tente faze-lo por conta propria (dica: comece com o
crculo ). N
ao se perca: queremos provar que o ponto Z est
a sobre a circunferencia .

Quem ja estudou homotetia j


a deve ter enxergado diversas homotetias entre as circunferencias, mas
muitos dos mais poderosos olmpicos do mundo foram derrotados por esse problema. De fato, dos 535
estudantes que participaram da IMO 2008, somente 13 resolveram (um deles fez 6 pontos) e 53 conseguiram
pelo menos um ponto. Isto quer dizer que mais de 90% dos estudantes zeraram o problema!
Isso e sinal de que esse problema deve ter algo novo para ser explorado. De fato, uma transformacao
geometrica que esteve em voga nos anos 80 e desapareceu nos anos 90 foi a homotetia. E ela voltou,
discretamente em 2007 e com tudo em 2008!
Vamos definir homotetia, ver algumas de suas propriedades e expandir as ideias envolvidas nessa transformacao.
1.

Homotetia: defini
c
ao

Voce vai ver que homotetia nada mais e do que fazer sombrinha. Aparecem muitos paralelismos, mas o
mais interessante sao as colinearidades que ir
ao aparecer. No incio parece magica; mas um bom matematico
sempre revela seus truques!
Vamos comecar com a definicao de homotetia com raz
ao positiva ou homotetia direta:

Defini
c
ao 1.1. Homotetia de uma figura F com centro O e raz
ao k, sendo k um n
umero real positivo, e
uma transformacao geometrica que associa a cada ponto P de F o ponto P sobre a semi-reta OP , de origem
O, tal que OP = k OP .

Talvez com vetores seja mais interessante: sendo O o centro da homotetia, o ponto P e transformado no

ponto P de modo que OP = k OP . Note que a homotetia e uma funcao que leva pontos do plano (ou do
espaco, se voce estiver trabalhando em dimensoes maiores) a pontos do plano (espaco). De fato, podemos
fazer P = (P ), tal que (P ) O = k (P O) (P ) = O + k (P O).
Com isso, podemos definir homotetias para k negativo tambem, obtendo as chamadas homotetias de
raz
ao negativa ou homotetias inversas:
Defini
c
ao 1.2. Homotetia de uma figura F com centro O e raz
ao k, sendo k um n
umero real negativo, e
uma transformacao geometrica que associa a cada ponto P de F o ponto P sobre a reta OP , de origem O,

tal que OP = k OP .

Note que imagens de homotetias inversas ficam de cabeca para baixo.


2.

Propriedades da homotetia

As principais propriedades de homotetias tem a ver com colinearidade e concorrencia. Algumas tem a ver
com paralelismo.
2.1.

2.2.

Colinearidade
O centro de homotetia, o ponto e seu transformado sao colineares. Em outras palavras, O, P e P = (P )
sao colineares. Isso decorre diretamente da definicao, mas homotetias n
ao vem de graca! Normalmente
as encontramos nos problemas e, com essa propriedade, obtemos pontos colineares.
Concorr
encia
O centro de homotetia pertence a todas as retas que ligam pontos a seus transformados. Em outras
palavras, O pertence a toda reta do tipo P P = P (P ). Novamente, uma propriedade que decorre diretamente da definicao (na verdade, e a mesma da colinearidade!), mas que aparece quando descobrimos
alguma homotetia.

2.3.

Paralelismo

A reta que liga dois pontos e paralela `a reta que liga os seus transformados. Em outras palavras, P Q
e P Q = (P )(Q) sao paralelas. A demonstracao desse fato vem da semelhanca entre OP Q e OP Q
(pelo caso LAL!).

Dois triangulos com lados respectivamente paralelos sao homoteticos. Para provar isso, sendo ABC
e DEF os triangulos com AB, DE, AC, DF e BC, EF respectivamente paralelos, use o teorema de
Desargues para provar que esses triangulos sao perspectivos.

Em particular, algumas figuras sao sempre semelhantes: os crculos! Com isso, temos a seguinte propriedade:
2.4.

Crculos
Dois crculos sao sempre homoteticos. Com excecao de crculos concentricos, eles admitem duas homotetias, uma direta e uma inversa. No caso de crculos disjuntos, os centros de homotetias sao faceis de
encontrar: sao as intersecoes das tangentes comuns internas (inversa) e das tangentes comuns externas
(direta).

Com isso, podemos resolver alguns problemas. Homotetia esteve bastante na moda na IMO durante o
incio dos anos 80, como voce vai ver nos exemplos e nos exerccios.
Exemplo 2.1.

Problema 5, IMO 1981. Tres crculos congruentes tem um ponto comum O e est
ao no interior de um
triangulo. Cada crculo e tangente a dois lados do triangulo. Prove que o incentro e o circuncentro do
triangulo e o ponto O sao colineares.

Resolu
c
ao
O nome do ponto dado n
ao e O por acaso: sejam A, B e C os centros dos tres crculos congruentes e
A B C o triangulo cujos lados tangenciam esses tres crculos. Note que os raios dos crculos congruentes
sao OA = OB = OC, isto e, O e circuncentro de ABC. Alem disso, das tangencias dos crculos com os
lados temos que AA , BB e CC sao as bissetrizes do triangulo A B C e se interceptam no incentro I do
triangulo.

As distancias de A e B a A B sao iguais aos raios dos crculos congruentes e sao, portanto, iguais.
Ent
ao AB e A B sao paralelos. Analogamente, AC e paralelo a A C e BC e paralelo a B C , de modo
que os triangulos ABC e A B C sao homoteticos. O centro de homotetia e I. Essa homotetia leva O ao
circuncentro O de A B C . Assim, I, O e O sao colineares.
Note que a dificuldade do problema foi achar a homotetia; depois bastou aplicar a propriedade de
colinearidade.
Exerccios
01. (Problema 2, IMO 1982) Seja A1 A2 A3 um triangulo escaleno com lados a1 , a2 e a3 (ai e o lado oposto
a Ai ). Seja Mi o ponto medio do lado ai e Ti o ponto onde o incrculo do triangulo toca o lado ai , para
i = 1, 2, 3. Seja Si o simetrico de Ti em relacao `a bissetriz interna do angulo Ai . Prove que as retas M1 S1 ,
M2 S2 e M3 S3 sao concorrentes.
02. (Problema 2, IMO 1983) Seja A um dos dois pontos de intersecao dos crculos C1 e C2 , de centros
O1 e O2 , respectivamente. Uma das tangentes comuns aos crculos toca C1 em P1 e C2 em P2 , e a outra
toca C1 em Q1 e C2 em Q2 . Seja M1 o ponto medio de P1 Q1 e M2 o ponto medio de P2 Q2 . Prove que
6 O1 AO2 = 6 M1 AM2 .
03. (Prova de Selecao 2008, Banco da IMO 2007) As diagonais do trapezio ABCD cortam-se no ponto P .
O ponto Q est
a na regi
ao determinada pelas retas paralelas BC e AD tal que 6 AQD = 6 CQB e a reta CD
corta o segmento P Q. Prove que 6 BQP = 6 DAQ.
3.

O Fen
omeno Homot
etico Circular

Algumas aplicacoes de certos teoremas sao t


ao conhecidos quanto os proprios. Para homotetias, e o caso com
o fenomeno homotetico circular, que mostra uma colinearidade bastante interessante envolvendo incrculo e
ex-incrculo.

Fen
omeno Homot
etico Circular. Seja ABC um triangulo e sejam K e L os pontos de tangencia do
incrculo e ex-incrculo relativo a A em BC. Ent
ao A, L e o ponto K diametralmente oposto a K no
incrculo sao colineares.
Demonstra
c
ao

Basta tracar a reta B C paralela a BC que tangencia o incrculo de ABC em K . Ent


ao os triangulos
ABC e AB C sao homoteticos com centro em A. Para terminar, o incrculo de ABC e ex-incrculo de
AB C , de modo que os pontos K e L sao correspondentes na homotetia e est
ao, portanto, alinhados com
A.
Vale a pena lembrar tambem que, na figura acima, BK = LC.
Exerccios
04. (Problema 4, IMO 1992) No plano, considere uma circunferencia C, uma reta L tangente `a circunferencia
e M um ponto da reta L. Encontre o lugar geometrico dos pontos P com a seguinte propriedade: existem
dois pontos Q, R da reta L tais que M e o ponto medio de QR e C e a circunferencia inscrita no triangulo
P QR.
4.

Composi
c
ao de homotetias

A principal inovacao na IMO 2008 no problema 6 foi explorar o seguinte fato:


Composi
c
ao de homotetias. Se 1 e uma homotetia de centro O1 e 2 e uma homotetia de centro O2
ent
ao a composicao de homotetias = 2 1 e uma homotetia de centro O, e O1 , O2 e O est
ao alinhados.
Au
nica excecao e quando a composicao e uma translacao.
Demonstra
c
ao
Utilizaremos vetores para provar esse fato.

Seja P um ponto qualquer e sejam k1 e k2 as raz


oes de homotetia de 1 e 2 , respectivamente. Ent
ao
1 (P ) = O1 + k1 (P O1 ) e, portanto,
(P ) = 2 1 (P ) = 2 (1 (P )) = O2 + k2 (1 (P ) O2 )
= O2 + k2 (O1 + k1 (P O1 ) O2 )
= k2 (1 k1 ) O1 + (1 k2 ) O2 + k1 k2 P

()

Primeiro, se e uma homotetia, ent


ao sua raz
ao e k1 k2 (as figuras sao multiplicadas por k1 e depois
por k2 ; ou seja, sao multiplicadas por k1 k2 ). Assim, para provarmos que e uma homotetia, temos que
provar que existe um ponto O tal que
(P ) = O + k1 k2 (P O) = (1 k1 k2 ) O + k1 k2 P

()

Comparando os coeficientes em () e () conclumos que (1 k1 k2 )O = k2 (1 k1 ) O1 + (1 k2 ) O2 .


1 +(1k2 )O2
Se k1 k2 = 1, e uma translacao (verifique!). Caso contrario, O = k2 (1k1 )O
e, como k2 (1 k1 ) +
1k1 k2
(1 k2 ) = k2 k1 k2 + 1 k2 = 1 k1 k2 , O e uma media ponderada de O1 e O2 . Em outras palavras, O
pertence `a reta O1 O2 .
Os partid
arios da geometria sintetica devem estar sentindo falta de uma demonstracao sintetica. Vamos
provar a parte da colinearidade sinteticamente.
Demonstra
c
ao sint
etica da colinearidade
Considere dois pontos P e Q e seus transformados P1 = 1 (P ), Q1 = 1 (Q), P2 = 2 (P1 ) = (P ) e
Q2 = 2 (Q1 ) = (Q).

Note que, das homotetias, P Q, P1 Q1 e P2 Q2 sao paralelos. Em termos projetivos, eles sao concorrentes
em um ponto do infinito. Isto quer dizer que os triangulos P P1 P2 e QQ1 Q2 sao perspectivos e podemos aplicar
o teorema de Desargues: as intersecoes entre lados correspondentes, P P1 QQ1 = {O1 }, P1 P2 Q1 Q2 = {O2 }
e P P2 QQ2 = {O} sao colineares.
4.1.

Detalhe t
ecnico

Geralmente, trabalhamos com homotetias sinteticamente, e aparecem homotetias diretas e inversas. Homotetias inversas multiplicam figuras por fatores negativos, de modo que a composicao de duas homotetias do
mesmo tipo e direta e a composicao de duas homotetias de tipos diferentes e inversa. Para facilitar, a homotetia inversa faz o papel do sinal de menos e a homotetia direta, do sinal de mais. Na composicao de
homotetias, seguimos a regra dos sinais da multiplicacao.
Agora estamos prontos para resolver o problema 6 da IMO 2008. Vamos reenunciar o problema e
resolve-lo.

Exemplo 4.1.
Problema 6, IMO 2008. Seja ABCD um quadril
atero convexo cujos lados BA e BC tem comprimentos diferentes. Sejam 1 e 2 as circunferencias inscritas nos triangulos ABC e ADC, respectivamente.
Suponhamos que existe uma circunferencia tangente `a reta BA de forma que A est
a entre B e o ponto
de tangencia, tangente `
a reta BC de forma que C est
a entre B e o ponto de tangencia, e que tambem seja
tangente `as retas AD e CD. Prove que as tangentes comuns exteriores a 1 e 2 se intersectam sobre .
Resolu
c
ao
Vamos comecar trabalhando com segmentos tangentes.

Temos BE = BF , AF = AG, CE = CH e DG = DH. Ent


ao AB = BF AF = BE AG =
BC + CE (AD + DG) = BC + CH AD DH = BC AD + (CH DH) = BC AD + CD =
AB + AD = BC + CD. Note que esse fato depende somente de ser tangente aos prolongamentos dos
lados do quadril
atero ABCD (guarde esse fato, ele pode ser u
til em outros problemas!). Isso implica
AC+ABBC
AC+CDAD
=

CK
=
AL.
2
2
Essa igualdade e simples, mas abre muitas portas para n
os! De fato, ela quer dizer que os ex-incrculos
3 e 4 relativos a AC dos triangulos ABC e ADC tocam AC em K e L, respectivamente. Isso nos d
a
muitas, mas muitas homotetias, e pelo menos duas oportunidades de utilizar o fenomeno homotetico circular!
Desenhemos as circunferencias:

Vamos compor homotetias para descobrir colinearidades, utilizando 3 e 4 como intermedi


arios!

21
41
24
1 . O centro K da homotetia (direta) 24 , o centro B da homotetia (direta)
1 e 2
4
2
41 e o centro Z da homotetia (direta) 21 est
ao alinhados. Isso quer dizer que Z pertence `a reta BK.

21
31
23
1 . O centro D da homotetia (direta) 23 , o centro L da homotetia (direta)
1 e 2
3
2
31 e o centro Z da homotetia (direta) 21 est
ao alinhados. Isso quer dizer que Z pertence `a reta DL.

Com isso, conclumos que Z e a intersecao de BK e DL.


Note que ate agora n
ao envolvemos o crculo nas homotetias. Agora e a hora, mas vamos provar
colinearidades de outra forma. Seja W a intersecao de BK e . Provaremos que W = Z, resolvendo o
problema.

Primeiro, note que a homotetia direta 4 que leva 4 a tem centro B e, portanto, leva K a W . Mais
ainda: como AC e tangente a 4 em K, a reta r paralela a AC que passa por W e tangente a , pois a reta
AC e levada a r por 4 .
Agora, considere a homotetia inversa 2 que leva a 2 . Essa homotetia tem centro em D, leva W a
T e r a s, que e paralela a r e AC e e tangente a 2 . Assim, D, W e T est
ao alinhados, ou seja, W pertence
`a reta DT .
Falta ainda identificar melhor o ponto T . Na verdade, ele e bem conhecido: como s e AC sao paralelos,
T e K sao diametralmente opostos. Podemos, assim, aplicar o fenomeno homotetico circular: D, T e L sao
colineares e L tambem pertence `
a reta DT .
Portanto D, L e W sao colineares, de modo que W pertence a DL. Como W pertence, por definicao, `a
reta BK, W e a intersecao de BK e DL, e so pode ser igual a Z.
Observacao: Note que a condicao AB 6= AC e importante para que as retas BK e DL n
ao coincidam.
Exerccios
05. (Banco da IMO 2007) O ponto P pertence ao lado AB do quadril
atero convexo ABCD. Seja o
incrculo do triangulo CP D e I o seu incentro. Suponha que e tangente aos incrculos dos triangulos AP D
e BP C em K e L, respectivamente. As retas AC e BD se encontram em E e as retas AK e BL se encontram
em F . Prove que os pontos E, I e F sao colineares.
06. (Romenia) Seja ABC um triangulo e a , b , c crculos dentro de ABC tangentes exteriormente duas
a duas, tais que a e tangente a AB e AC, b e tangente a AB e BC e c e tangente a AC e BC. Sejam
D o ponto de tangencia entre b e c , E o ponto de tangencia entre a e c e F o ponto de tangencia entre
a e b . Prove que as retas AD, BE e CF tem um ponto em comum.

07. (Ira) Sejam e o incrculo e o circuncrculo do triangulo ABC. toca BC, CA e AB em D, E e F


respectivamente. Os tres crculos a , b e c tangenciam em D, E e F , repesctivamente, e em K, L e
M , respectivamente.
(a) Prove que DK, EL e F M tem um ponto P em comum.
(b) Prove que o ortocentro do triangulo DEF pertence `a reta OP .
08. Seja uma circunferencia e A, B e C pontos em seu interior. Construa as seguintes tres circunferencias:
1 tangente a , AB e AC; 2 tangente a , AB e BC; 3 tangente a , AC e BC. Sendo C1 , C2 e C3 os
respectivos pontos de tangencia de 1 , 2 , 3 com , prove que AC1 , BC2 e CC3 passam por um mesmo
ponto.

Problemas estranhos de geometria


O ttulo original iria ser geometria combinatoria, mas notei que muitos dos problemas abordados e tecnicas
a serem mostradas aqui nada tem de Combinatoria. De fato, os meus problemas favoritos daqui nao tem
Combinatoria alguma!
Eventualmente vai aparecer um pouco de Combinatoria. So porque alguns problemas nao usam suas
tecnicas nao quer dizer que Combinatoria vai deixar de aparecer em Geometria.
1.

Onde est
ao os pontos?

Na grande maioria dos problemas de Geometria, o primeiro passo e fazer uma figura que representasse bem
a situacao. E se isso nao possvel? E se nao for possvel nem mesmo desenhar uma figura?
Nesses casos, se nao podemos dizer onde estao exatamente os pontos, nos pelo menos procuramos suas
possveis posicoes. Sendo um pouco mais especfico, verificamos em que regiao ou regioes esta o ponto.
Exemplo 1.1.
(Ibero 2007, Problema 6) Seja F a famlia de todos os hexagonos convexos H que satisfazem as seguintes
condicoes:
(a) os lados opostos de H sao paralelos;
(b) quaisquer tres vertices de H podem ser cobertos com uma faixa de largura 1.
Determine o menor n
umero real ` tal que cada um dos hexagonos da famlia F pode ser coberto com
uma faixa de largura `.
Nota: Uma faixa de largura ` e a regiao do plano compreendida entre duas retas paralelas que estao a`
distancia ` (incluindo ambas as retas paralelas).
Resolu
c
ao
A primeira impressao e de que ` = 1, mas nao seria sem graca se fosse isso mesmo? A verdade e que ` =
Basta tomar quase-quadrados:

2.

Note que se `
< 2 entao existe  tal que ` = 2 . Basta tomar suficientemente pequeno para que
a faixa de largura 2  nao cubra o quase-quadrado correspondente.

Vamos provar que ` = 2 e suficiente para cobrir todos os hexagonos de F. Note que, para tanto, basta
mostrar
que todo hexagono de F tem algum par de lados opostos paralelos com distancia menor ou igual a

2.

Seja H um hexagono de F. Considere um triangulo


ABC formadopor vertices alternados de H. Note
que se algum dos lados de ABC e menor ou igual a 2, uma faixa de 2 e suficiente para cobrir H, pois a
distancia entre cada par de lados opostos de H e menor ou igual a cada um dos lados de ABC.

Assim, suponha que todos os lados de ABC sao maiores do que 2 e que a sua menor altura seja relativa
a A. Isto quer dizer que a distancia de A a` reta BC e menor ou igual a 1. Com isso, podemos mostrar que
1 = cos 45 =
o angulo 6 BAC e obtuso, pois, sendo H o pe da altura relativa a A, cos 6 HAB = AH
AB <
2

6 HAB > 45 e, analogamente, 6 HAC > 45 .

A nossa meta agora e descobrir onde pode estar o vertice A0 , oposto a A em H. Para isso, vamos estudar
a condicao (b) do enunciado. Note que ela e equivalente a dizer que todo triangulo determinado por vertices
de H tem sua menor altura menor ou igual a 1.
Suponhamos, entao, que sabemos a posicao de dois vertices X e Y do hexagono H e queiramos saber
onde poderiam estar os outros vertices de H. Seja Z um desses vertices. O interessante e que nao sabemos
qual e a menor altura, entao devemos pensar nas tres possibilidades (que na verdade sao duas, ja que duas
delas sao analogas):

A menor altura e relativa a Z. Entao a distancia de Z a XY e menor ou igual a 1. Sendo o conjunto dos
pontos a uma distancia fixada de uma reta igual a um par de retas paralelas, conclumos que Z pode
estar na faixa formada por duas retas r e s a uma distancia 1 de XY , excetuando a propria reta XY :

A menor altura e relativa a X. Entao a distancia de X a Y Z e menor ou igual a 1. Agora temos uma
situacao mais interessante, pois o ponto variavel e Z; como medir a distancia de um ponto fixado a uma

reta variavel? O truque aqui e considerar um crculo C de raio 1 com centro em X. Se a reta e tangente
a C, entao a distancia de X a essa reta e 1; se for secante, e menor que 1; se for exterior, e maior que
1. Considerando ainda que as retas em questao devem passar pelo outro ponto fixo Y , temos a nossa
resposta:

A menor altura e relativa a Y . Esse caso e analogo ao anterior.

Assim, dados os pontos X e Y do hexagono, os demais vertices pertencem a` regiao destacada na figura
a seguir:

Para facilitar, chamaremos tal regiao de XY -regiao.


Com isso, podemos continuar o problema. Queremos encontrar as possveis posicoes do ponto A 0 , oposto
a A no hexagono. Ele deve pertencer a` AB-regiao e a` AC-regiao, ou seja, na intersecao dessas duas regioes.
Alem disso, como o hexagono e convexo, C e o vertice C 0 , oposto a C, estao em lados opostos em relacao a
AB e AC 0 e paralelo a A0 C, A0 tambem pertence a` regiao delimitada por CR k AB, BC e, analogamente,
BQ k AC.

0
Se A0 esta no interior do angulo 6 CBT , ent
ao a distancia de A ao lado0 A B e menor ou igual a 1 e e
possvel cobrir H com uma faixa de largura 1 < 2, pois a distancia entre A B e o lado oposto, que contem
A, e menor ou igual a 1; da mesma forma, se A0 esta no interior do angulo 6 BCS, a distancia de A ao lado

A0 C e menor ou igual a 1 e e possvel cobrir H com uma faixa de largura 1. Caso contrario, AA 0 < , pois

6 M AP > 90 , e
e possvel cobrir H com uma faixa de largura AN < 2.

Na verdade, pode-se provar que esse u


ltimo caso nao pode ocorrer: basta considerar o maior entre os
angulos 6 A0 AB e 6 A0 ACe provar (por eliminacao, e usando o fato de que o seno de um angulo maior do
que 45 e maior do que 2/2) que a distancia de A a A0 B ou A0 C (o que participar do maior angulo) e
menor ou igual a 1, considerando o triangulo AA0 B (ou AA0 C).

Exerccios
01. (Hungria 1998) Seja P um pol
gono convexo com lados de medidas inteiras e permetro mpar. Prove
que a area de P e maior ou igual a 3/4.
02. (Cone Sul 1996, Problema 6) Achar todos os n
umeros inteiros n 3 tais que exista um conjunto S n
formado por n pontos do plano que satisfacam as duas condicoes seguintes:
(1) Tres pontos quaisquer nao sao colineares.
(2) Nenhum ponto se encontra no interior do crculo cujo diametro tem por extremos dois pontos quaisquer
de Sn .
NOTA: Os pontos da circunferencia nao sao considerados interiores ao crculo.
03. (OBM 2005, Nvel 3, Problema 3) Dizemos que um quadrado esta contido em um cubo quando todos
os seus pontos estao nas faces ou no interior do cubo. Determine o maior ` > 0 tal que existe um quadrado
de lado ` contido num cubo de aresta 1.
04. (Cone Sul 2005, Problema 6) No plano cartesiano tracamos circunferencias de raio 1/20 com centros
em cada ponto de coordenadas inteiras. Mostre que qualquer circunferencia de raio 100 que se trace no plano
intersecta pelo menos uma das circunferencias pequenas.
2.

Cobrindo figuras

Problemas desse tipo podem ser resolvidos com uma variedade bastante grande de tecnicas. Uma mistura
de casos extremos, casa dos pombos e saber construir exemplos geralmente funciona bem.
Exemplo 2.1.
(USAMO 2007, Problema 2) Um reticulado no plano cartesiano consiste em todos os pontos (m, n), onde m
possvel cobrir todos os pontos do reticulado com uma famlia infinita de crculos cujos
e n sao inteiros. E
interiores nao se sobrepoem se cada crculo da famlia tem raio maior ou igual a 5?
Resolu
c
ao
A resposta e nao (os crculos sao muito grandes, nao?). Para isso, suponha que existe uma cobertura desse
tipo e seja C omaior crculo que nao se sobrepoe com algum crculo da famlia. Entao o seu raio r deve ser
menor do que 2/2; caso contrario, C cobriria um ponto do reticulado, que nao seria coberto pela famlia.

Alem disso, C tangencia pelo menos tres crculos C1 , C2 , C3 da famlia. Sejam O, O1 , O2 e O3 os


centros dos crculos C, C1 , C2 , C3 , respectivamente. Entao um dos angulos 6 O1 OO2 , 6 O2 OO3 , 6 O3 OO1 e
menor ou igual a 120 . Suponha, sem perdas, que e 6 O1 OO2 . Entao, pela lei dos co-senos,
O1 O22 OO12 + OO22 + OO1 OO2

Sejam r1 e r2 os raios de C1 e C2 , respectivamente. Entao O1 O2 r1 + r2 , OO1 = r + r1 e OO2 = r + r2 .


Substituindo, obtemos
(r1 + r2 )2 (r + r1 )2 + (r + r2 )2 + (r + r1 )(r + r2 ) 12r 2 (r1 3r)(r2 3r)
Mas r <

2/2 e r1 e r2 sao maiores ou iguais a 5, de modo que

12r2 (5 3r)2 2 3r 5 3r r

Um calculo rapido mostra que

5
3+2 3

>

2
2 ,

3+2 3

e o problema acabou.

Exerccios
05.

Tres crculos de raio r cobrem um crculo de raio 1. Encontre o menor valor de r.

06. (OBM 2002, Nvel 3, Problema 5) Temos um n


umero finito de quadrados, de area total 4. Prove que e
possvel arranja-los de modo a cobrir um quadrado de lado 1.
permitido sobrepor quadrados e parte deles pode ultrapassar os limites do quadrado a ser
Obs: E
coberto.
3.

Princpio do extremo

Considerar a maior (ou menor) distancia ou algum triangulo de area maxima (ou mnima) ou . . . maximo
(ou mnimo) pode ser bastante u
til.
Exemplo 3.1.
(OBM 2007, Nvel 3, Problema 3) Sao dados n pontos no plano, os quais sao os vertices de um polgono
convexo. Prove que o conjunto das medidas dos lados e das diagonais do polgono tem pelo menos bn/2c
elementos distintos.
Resolu
c
ao
Talvez esse nao seja o exemplo mais simples no assunto, mas ele envolve tantas ideias interessantes que
merece ser resolvido.
Primeiro, considere dois pontos A e B do polgono cuja distancia e maxima. Tome B de modo que AB
separe o polgono em dois polgonos, um deles com AB como u
nica distancia maxima.
Em cada um desses dois polgono vamos aplicar o seguinte
Lema 3.1. Seja A1 A2 . . . Ak um polgono convexo tal que a maior distancia entre dois de seus vertices,
incluindo diagonais, e A1 Ak . Entao esse polgono tem k 2 distancias diferentes; caso A1 Ak seja a u
nica
distancia maxima, entao ha k 1 distancias diferentes.
Demonstra
c
ao
Sejam Ap e Aq , 1 < p < q < k dois vertices do polgono. Vamos provar que, para quaisquer m e n com
p < m n < q um dos segmentos Ap An , Aq Am e menor do que Ap Aq . Em seguida, conseguiremos uma
seq
uencia de k 2 distancias diferentes.
Como conseguir distancias menores? Ou, de modo mais geral, como comparar segmentos? Muitas vezes
e melhor transferir tudo para angulos, para que possamos fazer. . . isso mesmo, um arrastao!

Sejam = 6 Am A1 Ak , 1 = 6 A1 Am Ak , 2 = 6 Ap Am Aq , 3 = 6 Aq Ap Am , A a intersecao de Ap Aq e
A1 Am (note que, como o polgono e convexo, A esta no interior do segmento A p Aq ) e 4 = 6 Am AAq .

Suponha que Ap Aq Am Aq . Entao, no triangulo Am Ap Aq , 2 3 . Alem disso, pelo teorema do


angulo externo no triangulo AAp Am , 3 < 4 . Ademais, 1 < 2 e, sendo A1 Ak a maior distancia de todas
(e esse e o passo decisivo da demonstracao e mostra o poder do princpio do extremo), no triangulo A 1 Am Ak ,
< 1 . Logo
< 1 < 2 3 < 4 = < 4
Definindo os s analogamente e supondo que Ap Aq An Ap , obtemos < 4 . Porem, observando os
quadrilateros A1 Ak An Am e ABAn Am , temos que + + 6 A1 Am An + 6 Ak An Am = 4 + 4 + 6 AAm An +
6 BAn Am = 360 = + = 4 + 4 . Mas

<
4

= + < 4 + 4 ,

< 4
contradicao.

O caso em que m = n fica a cargo do leitor.


Para terminar, basta fazer uma especie de zigue-zague. Comece com A 2 Ak1 , que e menor do que
A1 Ak (por que? veja o proximo exerccio!). Pelo que acabamos de provar, A2 Ak2 ou A3 Ak1 e menor do
que A2 Ak1 . Oba, mais uma distancia! Suponha, por exemplo, que A3 Ak1 seja menor. Entao, aplicando o
nosso fato de novo, A4 Ak1 ou A3 Ak2 e menor do que A3 Ak1 . Continuamos assim, ate acabar o polgono,
e assim conseguimos (conte!) k 2 distancias diferentes.
No caso em que A1 Ak e a u
nica distancia maxima, fica para voce provar (use o poder do arrastao
novamente!) que, no quadrilatero A1 A2 Ak1 Ak , uma das diagonais e menor do que A1 Ak (bem, isso e
imediato ^)
e maior do que A2 Ak1 (nisso voce vai ter que trabalhar um pouquinho mais ^),
de modo que
ganhamos mais uma distancia, totalizando k 1.
Agora, vamos terminar o problema. Lembre que cortamos o polgono original do problema em dois
por uma diagonal AB com medida maxima. Suponha que os polgonos obtidos tenham k + 1 e n k + 1
lados, sendo que o de k + 1 lados tem a distancia maxima u
nica. Nele, obtemos (k + 1) 1 = k distancias
diferentes, e no outro, (n k + 1) 2 = n k 1. Entao conseguimos d = max{k, n k 1} distancias.
n
Mas d k+(nk1)
= n1
2
2 b 2 c.
Exerccios
07. Seja S um conjunto finito de pontos. Prove que se A, B, C, D sao pontos distintos de S tais que
AB e CD sao ambos segmentos de distancia maxima, entao esses segmentos se cortam em seus respectivos
interiores.
08. (Olimpada de Maio 2003) Beto marcou 2003 pontos verdes no plano, de maneira que todos os triangulos
com seus tres vertices verdes tem area menor que 1.
Demonstre que os 2003 pontos verdes estao contidos num triangulo T de area menor que 4.

09. (Um classico! O Teorema de Sylvester) Dados n pontos, nao todos colineares, prove que existe uma
reta que passa por exatamente dois desses pontos.
10. (Cone Sul 2001, Problema 3) Tres triangulos acutangulos estao inscritos em uma mesma circunferencia,
de modo que seus vertices sao nove pontos distintos. Demonstre que se pode escolher um vertice de cada
triangulo de maneira que os tres pontos escolhidos determinem um triangulo cujos angulos sejam menores
que ou iguais a 90 .
11. (Cone Sul 2001, Problema 4) Um polgono de area S esta contido no interior de um quadrado de lado
a. Demonstre que ha pelo menos dois pontos do polgono que estao separados por uma distancia maior que
ou igual a S/a.
12. (Vinganca Olmpica 2005, Problema 5) Encontre todos os conjuntos finitos X de pontos no plano, nao
todos colineares, satisfazendo a seguinte condicao: para quaisquer duas circunferencias distintas, cada uma
passando por tres pontos distintos de X, a intersecao delas tambem esta contida em X.
13. (OBM 1996, Problema 2) Existe um conjunto finito de n > 2 pontos no plano tais que nao ha tres
pontos colineares e o circuncrculo de quaisquer tres pontos pertence ao conjunto?

4.

Fecho convexo

Fecho convexo de um conjunto S de pontos (finito ou infinito) limitado no plano (poderia ser do espaco ou
de figuras com mais dimensoes ainda) e a menor figura convexa que contem S. No caso do plano, e figura
obtida quando soltamos um elastico esticado em torno de S.
Exemplo 4.1.
(IMO 2002, Problema 6) Sejam 1 , 2 , . . . , n crculos de raio 1 no plano, onde n 3. Seus centros sao O1 ,
O2 , . . . , On , respectivamente.
Suponha que nao exista reta que intercepte mais que dois dos crculos. Prove que

1i<jn

(n 1)
1

Oi Oj
4

Resolu
c
ao
O que e isso? Contas com segmentos de um lado e do outro? O que fazer?
A primeira ideia e, na verdade, transformar tudo em angulos. Seja ij o angulo determinado entre as
tangentes internas a i e j .

Veja que sen

Assim,

1
Oi Oj

ij
2

1
Oi Oj /2

sen /2
2

<

4,

2
Oi Oj .

A figura a seguir mostra que, para todo x real positivo, sen x < x.

de modo que basta provar que

1i<jn

ij
(n 1)

4
4

ij (n 1)

1i<jn

Voltemos para os ij s. Eles tambem aparecem nos proprios crculos, como ilustramos na figura a seguir,
tracando as tangentes externas (que sao paralelas):

A condicao de nao haver retas que interceptem mais do que dois crculos se traduz em os ij /2s nao se
sobreporem nas figuras. A regiao hachurada indica em que regioes as circunferencias diferentes de i e j
nao podem estar.

Assim, a soma de todos os ij s e menor ou igual a` metade da soma de todos os arcos (veja que cada
ij /2 aparece quatro vezes, totalizando 2ij ), que em princpio e n.
a que o fecho convexo entra! soltando
Faltou pouco! Mas sera que nao podemos melhorar essa conta? E
o elastico, vemos que alguns arcos nao sao contados:

A soma desses arcos e exatamente a soma dos angulos externos de um polgono, que e 2. Com isso, a
metade da soma de todos os arcos e menor ou igual a n = (n 1), e acabou.
Exerccios
14.

Dados n pontos no plano, prove que tres deles determinam um angulo menor ou igual a 180 /n.

5.

Dist
ancias e n
umeros reais

Muitos problemas exploram distancias, e em alguns deles queremos saber se eles satisfazem certas condicoes.
Exemplo 5.1.
(Ibero 1996, Problema 6) Tem-se n pontos distintos A1 , A2 , . . . , An no plano e a cada ponto Ai se associa
um n
umero real i diferente de zero, de maneira que Ai A2j = i + j , para todos i, j com i 6= j. Demonstre
que
(a) n 4
(b) Se n = 4, entao

1
1

1
2

1
3

1
4

= 0.

Resolu
c
ao
Distancias ao quadrado! Que teorema envolve distancias ao quadrado? Uma dica: o nome do teorema
comeca com Pit e termina com agoras. Com o seu auxlio, provamos o seguinte
Lema 5.1. Se A, B, C e D sao pontos no plano entao AB 2 + CD2 = AD 2 + BC 2 se, e somente se, AC e
BD sao perpendiculares.
Demonstra
c
ao
Seja B 0 e D0 as projecoes de B e D sobre a reta AC, respectivamente. Suponha que B 0 esta mais a` esquerda

de D0 . Seja B 0 D0 = k.

Aplicando o teorema de Pitagoras quatro vezes, obtemos


AB 2 = a2 + b2

AD2 = (a + k)2 + d2

CD2 = c2 + d2

BC 2 = (c + k)2 + b2

Assim,
AB 2 + CD2 = AD 2 + BC 2 a2 + b2 + c2 + d2 = (a + k)2 + b2 + (c + k)2 + d2
2k(a + c + k) = 0 2k AC = 0 k = 0,
de modo que as projecoes de B e D sobre AC devem coincidir, provando que a igualdade ocorre se, e somente
se, BD e perpendicular a AC.
Com isso, o problema fica simples. Suponha que n 4, de modo que existam A 1 , A2 e A3 . Note que,
sendo i > 3, A1 A2i + A2 A23 = 1 + i + 2 + 3 = A1 A22 + A3 A2i , e pelo lema, A2 Ai e perpendicular a A1 A3 ,
claro que A2 nao e mais especial do que A1 e A3 , assim,
isto e, Ai pertence a` altura por A2 de A1 A2 A3 . E
analogamente Ai tambem pertence a` altura por A1 de A1 A2 A3 . Deste modo, Ai esta na intersecao das duas
alturas, ou seja, seu ortocentro. Isso vale para todo ponto Ai diferente de A1 , A2 e A3 . Mas como so existe
um ortocentro, so pode haver um ponto a mais, ou seja, n 4. Isso resolve o item a.
Vamos para o item b. Primeiro, considere tres pontos Ai , Aj e Ak . Entao


Ai A2j = i + j


Ai A2k = i + k

A A2 = +
j k
j
k
Mas, pela lei dos co-senos,


A A2 + Ai A2k Aj A2k

i = i j

2

2
2
2

A
A
+
A
j Ak A i Ak
j = i j
2


Ai A2k + Aj A2k Ai A2j

k =
2

Ai A2j +Ai A2k Aj A2k


2

= Ai Aj Ai Ak cos 6 Aj Ai Ak . Assim,


i = Ai Aj Ai Ak cos 6 Aj Ai Ak


j = Ai Aj Aj Ak cos 6 Ai Aj Ak

k = Ai Ak Aj Ak cos 6 Ai Ak Aj

Vamos supor que A4 e ortocentro do triangulo acutangulo A1 A2 A3 (na verdade, cada ponto e ortocentro
do triangulo determinado pelos tres outros pontos). Note que o triangulo nao pode ser retangulo, o que prova
que nenhum dos s pode ser zero.

Note que 6 A2 A4 A3 = 6 A4 A2 A1 + 90 , de modo que cos 6 A4 A2 A1 = sen 6 A2 A4 A3 . Deste modo,


isolando os co-senos na relacao acima e substituindo na relacao fundamental cos 2 6 A2 A4 A3 +sen2 6 A2 A4 A3 =
1, obtemos

2 
2
4
2
+
=1
A2 A4 A 3 A4
A2 A4 A 1 A2
Como Ai A2j = i + j , tirando o mnimo e substituindo obtemos
24 (1 + 2 ) + 22 (3 + 4 ) = (1 + 2 )(3 + 4 )(2 + 4 )
Dividindo tudo por 1 22 3 24 ,
1
2 3

1
1
+
1
2

1
1 4

1
1
+
3
4

1
1
+
3
4

1
1
+
1
2



1
1
+
3
4



1
1
+
2
4

Separando o segundo membro:


1
2 3

1
1
+
1
2

1
1 4

1
2

1
1
+
1
2

Para que isso? Ora, para fatorar! Colocando


membro, obtemos

1
1
2 ( 1


+

1
1
+
3
4

1
2 )

1
1
4 ( 3

1
4

1
4 )

1
1
+
1
2



1
1
+
3
4

em evidencia no segundo







1
1
1
1
1
1
1
1
1
1
1
1
+
=0
+
+

+
2 1
2
3
4
3
4 1
2
1
3
4




1
1
1
1
1
1

+
+
+
=0
2 4 1
2
2 4 3
4


1
1
1
1
1
+
+
+

=0
2 4 1
2
3
4
1
1
1
1
+
+
+
=0

1
2
3
4

Exerccios
15. (Ibero 1997, Problema 6) Seja P = {P1 , P2 , . . . , P1997 } um conjunto de 1997 pontos no interior de um
crculo de raio 1, sendo P1 o centro do crculo. Para cada k = 1, . . . , 1997 seja xk a distancia de Pk ao ponto
de P mais proximo a Pk e distinto de Pk . Demonstrar que
x21 + x22 + + x21997 9
16. (Ibero 1998, Problema 5) Encontrar o maior valor possvel n para que existam pontos distintos P 1 , P2 ,
P3 , . . . , Pn no plano, e n
umeros reais r1 , r2 , . . . , rn de modo que a distancia entre quaisquer dois pontos
diferentes Pi e Pj seja ri + rj .
17. (IMO 1995, Problema 3) Determine todos os inteiros n > 3 tais que existem n pontos A 1 , A2 , . . . , An
no plan sem que haja tres deles colineares e n
umeros reais r1 , r2 , . . . , rn tais que, para todos i, j, k distintos,
a area do triangulo Ai Aj Ak e ri + rj + rk .
6.

Esse problema
e de geometria??

Muitas vezes ideias geometricas sao u


teis em problemas que aparentemente nao sao de Geometria. Como
em Combinatoria, por exemplo.
Exemplo 6.1.
(Cone Sul 1998, Problema 6) O prefeito de uma cidade deseja estabelecer um sistema de transportes com
pelo menos uma linha de onibus, no qual:
(i) cada linha passe exatamente por tres paradas;
(ii) cada duas linhas distintas tenham exatamente uma parada em comum;
(iii) para cada duas paradas de onibus distintas exista exatamente uma linha que passe por ambas.
Determine o n
umero de paradas de onibus da cidade.
Resolu
c
ao
Vamos representar linhas de onibus por uma reta e paradas por pontos. Uma linha passa por uma parada
se, e somente se, a reta correspondente passa pelo ponto correspondente. Reescrevendo as tres condicoes do
enunciado em termos de retas e pontos, temos:
(i) cada reta passa exatamente por tres pontos;
(ii) cada duas retas distintas tem exatamente um ponto em comum;
(iii) por dois pontos passa exatamente uma reta.
Assim, podemos desenhar (isso mesmo!) as possibilidades. Uma e ter exatamente uma linha (cidade
pequena essa, nao?), totalizando tres pontos A, B e C:

Agora, pode haver mais pontos. Considere um ponto D fora da reta. Por A e D passa uma reta, e
essa reta nao pode ser ABC, pois ela nao teria tres pontos; essa nova reta nao pode passar por B ou por C,
pois duas retas tem exatamente um ponto em comum. Assim, a reta AD precisa conter um novo ponto E.

Analogamente, obtemos as retas BDF e CDG.

Sera que pode haver mais um ponto H? Vejamos a reta DH. Ela deve cortar a reta ABC em um de
seus pontos, mas nenhum deles serve, ja que ja temos as retas AD, BD e CD e se um dos pontos A, B ou
C pertencesse a DH entao essa reta e AD, BD ou CD tem dois pontos de intersecao (D e um dos pontos
A, B, C). Entao nao podemos mais ter pontos. Completando as linhas, podemos obter o seguinte exemplo,
com 7 paradas (uma das linhas e circular):

Com um pouquinho mais de esforco voce pode provar que o u


nico exemplo para 7 paradas e esse.
Deste modo, a cidade tem 3 ou 7 paradas.
Exerccios
18. (OBM 2002, Nvel 3, Problema 6) Arnaldo e Beatriz se comunicam durante um acampamento usando
sinais de fumaca, a`s vezes usando uma nuvem grande, a`s vezes uma pequena.
No tempo disponvel antes do cafe da manha, Arnaldo consegue enviar uma seq
uencia de 24 nuvens.
Como Beatriz nem sempre consegue distinguir uma nuvem pequena de uma grande, ela e Arnaldo fizeram
um dicionario antes de ir para o acampamento. No dicionario aparecem N seq
uencias de 24 tamanhos de
nuvem (como por exemplo a seq
uencia P GP GP GP GP GP GGP GP GP GP GP GP , onde G significa nuvem
grande e P significa nuvem pequena). Para cada uma das N seq
uencias, o dicionario indica seu significado.
Para evitar interpretacoes erradas, Arnaldo e Beatriz evitaram incluir no dicionario seq
uencias parecidas.
Mais precisamente, duas seq
uencias no dicionario sempre diferem em pelo menos 8 das 24 posicoes.
Demonstre que N 4096.
Dica: pense em crculos! Defina distancia entre duas seq
uencias como a quantidade de posicoes com
nuvens de tamanhos diferentes nas duas seq
uencias. Por incrvel que pareca, essa distancia satisfaz a desigualdade triangular e podemos definir crculos de raio r e centro em uma seq
uencia s como as seq
uencias

com distancia menor ou igual a r de s. A ideia e, entao, pensar no fato de que crculos com centros distantes
nao podem se intersectar.
19. (IMO 1996, Problema 6) Sejam n, p, q inteiros positivos com n > p + q. Sejam x 0 , x1 , . . . , xn inteiros
que verificam as seguintes condicoes:
(a) x0 = xn = 0, e
(b) Para cada i, 1 i n, tem-se que
xi xi1 = p ou xi xi1 = q.
Demonstrar que existe um par (i, j) 6= (0, n), tal que xi = xj .
Dica: Primeiro mostre que n = k(p + q), k inteiro maior que 1. Em seguida, desenhe um quadriculado
kp kq e de um passo para a direita se xi xi1 = p e um passo para cima se xi xi1 = q. O que
significa xi = xj nesse quadriculado? Se voce conseguiu a resposta, a demonstracao termina ligando pontos!
7.

Refer
encias bibliogr
aficas

[1] Quer fontes de problemas para estudar? Primeiro, estude um pouco de ingles; depois, visite os seguintes
sites:
http://www.kalva.demon.co.uk/
http://www.mathlinks.ro/
claro, nao podemos esquecer o site da OBM (esse em portugues!):
[2] E
http://www.obm.org.br/
[3] Os colegas e amigos Davi Maximo e Samuel Feitosa escreveram um belssimo artigo na Eureka! 25:
Problemas sobre Pontos.

Geometria do Tri
angulo: fatos e problemas
Esse artigo envolve fatos n
ao t
ao conhecidos sobre triangulos como o teorema de Miquel, conjugados isogonais
e triangulos pedais, que sao u
teis em alguns problemas de Olimpada.
1.

O Teorema de Miquel

Comecamos com o teorema em si, que e um dos varios pequenos milagres dos chamados quadril
ateros
completos (veja um pouco mais desses milagres nos exerccios!), que sao os quadril
ateros conhecidos unidos
com as retas que contem os lados. Isto e, um quadril
atero completo e a uniao de quatro retas em vez de
quatro segmentos.
Teorema de Miquel. Sejam a, b, c, d quatro retas coplanares, de modo que n
ao h
a duas paralelas nem
tres concorrentes. Os circuncrculos dos quatro triangulos determinados pelas quatro retas passam por um
mesmo ponto, denominado ponto de Miquel das quatro retas.

Demonstra
c
ao
Seja M a intersecao dos circuncrculos de CEF e BDF na figura acima. Ent
ao 6 M EA = 6 M EC =

6
6
6
6
6
6
180 M F C = BF M = BDM = 180 ADM , de modo que M EA + ADM = 180 e, portanto,
M DAE e inscritvel. Isso quer dizer que M pertence ao circuncrculo de ADE. Analogamente, prova-se que
M pertence ao circuncrculo de ABC.
Vamos resolver, a ttulo de exemplo, o problema 6 da olimpada norteamericana de 2006.
Exemplo 1.1.
(USAMO 2006, Problema 6) Seja ABCD um quadril
atero e E e F pontos sobre os lados AD e BC, respectivamente, tal que AE/ED = BF/F C. A semirreta F E corta as semirretas BA e CD em S e T ,
respectivamente. Prove que os circuncrculos do triangulos SAE, SBF , T CF e T DE passam por um
mesmo ponto.
Resolu
c
ao
Ao fazer a figura, voce provavelmente vai notar uma certa semelhanca com a figura anterior.

Queremos provar que os pontos de Miquel de ADT S e BCT S coincidem! Isso n


ao e difcil, na verdade:
seja M a intersecao dos circuncrculos de SAE e SBF . Mostraremos que os circuncrculos de T ED e T F C
tambem passa por M .
Um arrast
ao e uma semelhanca d
ao conta do recado: primeiro, note que 6 AM E = 6 ASE = 6 BSF =
BM F e 6 M EA = 6 M SA = 6 M SB = 6 M F B. Ent
ao os triangulos AM E e BM F sao semelhantes,
e da igualdade AE/ED = BF/F C os triangulos M AD e M BC sao semelhantes tambem. Uma rapida
verificacao mostra que M AB e M DC tambem sao semelhantes: de fato, 6 AM B = 6 DM B 6 DM A =
6 DM C + 6 CM B 6 DM A = 6 DM C (pois como M AD e M BC s
ao semelhantes ent
ao 6 DM A = 6 CM B)
AM
= DM
(novamente
da
semelhan
c
a).
e BM
CM
6

Voce pode imaginar que o triangulo M AD gira em torno de M e, apos um acerto de escala, e
transformado no triangulo M BC. Isso e uma transformacao geometrica conhecida como roto-homotetia de
centro M . Assim, A e levado em B e D e levado em C. Note que a semelhanca obtida anteriormente envolve
o centro de roto-homotetia M , os pontos e suas imagens na transformacao. Isso na verdade sempre acontece
(e uma das semelhancas autom
aticas).
Agora podemos terminar o problema: da semelhanca entre M AB e M DC, os angulos externos 6 M DT
e 6 M AS sao congruentes. Como M pertence ao circuncrculo de SAE, 6 M AS = 6 M ES = 6 M ET , ou
seja, 6 M DT = 6 M ET , o que significa que M EDT e cclico e, portanto, M pertence ao circuncrculo de
T ED. Utilizando outra semelhanca autom
atica, entre M EF e M DC (pois e, E e levado em F !), prova-se
que M pertence tambem ao circuncrculo de T F C.
Note que se U e a intersecao de AB e CD, ent
ao pelo teorema de Miquel M tambem pertence ao
circuncrculo de ST U . Ent
ao na verdade cinco crculos passam pelo ponto M !
A seguinte versao do teorema de Miquel tambem e u
til:
Teorema de Miquel para tri
angulos. Seja ABC um triangulo e D, E, F pontos sobre as retas BC,
CA, AB, respectivamente. Ent
ao os circuncrculos de AEF , BF D e CDE tem um ponto em comum. Esse

ponto tambem e chamado de ponto de Miquel.

Demonstra
c
ao
Seja M a segunda intersecao dos circuncrculos de AEF e BF D. Ent
ao
180 6 CEM .
6

CDM =
6

BF M =
6

AEM =

Exerccios
01. Demonstre o teorema de Miquel para quadril
ateros utilizando o teorema de Miquel para triangulos.
02. Seja ABCDE um pent
agono convexo e F , G, H, I, J as intersecoes dos prolongamentos de EA, AB,
AB, BC, BC, CD, CD, DE e DE, EA, respectivamente. Prove que as segundas intersecoes dos circuncrculos
de ABF, BCG, BCG, CDH, CDH, DEI, DEI, EAJ e EAJ, ABF pertencem a uma mesma circunferencia.
03. Considere um quadril
atero completo. Seja M o seu ponto de Miquel. Prove que:
(a) os circuncentros dos quatro triangulos determinados pelo quadril
atero e M est
ao sobre uma mesma
circunferencia.
(b) as projecoes ortogonais de M sobre as quatro retas do quadril
atero pertencem a uma mesma reta r;
alem disso, M e o u
nico ponto do plano com essa propriedade.
(c) os ortocentros dos quatro triangulos pertencem a uma mesma reta s.
(d) as retas r e s sao paralelas, e a distancia de M a r e metade da distancia de M a s.
04. Seja ABCD um quadril
atero convexo e X e Y as intersecoes dos lados opostos AD e BC e AB e CD,
respectivamente. Prove que os pontos medios de AD, BC e XY sao colineares.
Observacao: a reta que passa pelos tres pontos e a reta de Gauss do quadril
atero completo.
2.

Conjugados isogonais

A ideia de conjugado e fazer uma associacao entre objetos. Objetos conjugados supostamente tem propriedades semelhantes. Isso e bastante comum em equacoes: se um n
umero e raiz, ent
ao o conjugado
tambem e raiz. Em geometria, tambem existe a ideia de conjugado. De fato, dado um triangulo, cada ponto
tem um conjugado isogonal e um conjugado isotomico. Aqui, trataremos somente de conjugados isogonais.

Defini
c
ao 2.1. Dado um triangulo ABC, o conjugado isogonal em relacao a ABC de um ponto T do
plano de ABC e obtido refletindo as retas T A, T B e T C em relacao `as bissetrizes internas de ABC que
passam por A, B e C, respectivamente. As retas resultantes sao concorrentes no isogonal T 1 de T .
A seguir, as linhas pontilhadas sao as bissetrizes, e as cevianas cinzas sao as reflex
oes das cevianas
pretas.
A

T -1

T
B

O fato de que as retas isogonais sao concorrentes e extremamente importante, tanto que sera enunciado
novamente.
Teorema fundamental dos conjugados isogonais. Dados um triangulo e tres retas que passam pelos
respectivos vertices e concorrem em um ponto P , as retas isogonais a elas, obtidas atraves da reflex
ao em
relacao `a bissetriz interna correspondente, sao concorrentes no conjugado isogonal P 1 de P .
Demonstra
c
ao
Por que as cevianas cinzas sao concorrentes? Isso decorre de duas aplicacoes do teorema de Ceva trigonometrico: primeiro com as cevianas concorrentes em T e depois, com as cevianas concorrentes em T 1 , que
formam os mesmos
angulos que as outras cevianas, porem no sentido contrario.
Na verdade, pode ocorrer de as tres cevianas serem paralelas. Isso ocorre se, e somente se, T est
a sobre
o circuncrculo de ABC; nesse caso, pensamos projetivamente, ou seja, o conjugado isogonal e um ponto do
infinito.
2.1.

Para que servem isogonais?

O que e mais u
til em conjugados isogonais e simplesmente que as cevianas sao reflex
oes umas das outras em
relacao `as bissetrizes, e isso costumam levar a algumas igualdades entre angulos um pouco mais difceis de
obter ou mesmo de se imaginar com contas.
Exemplo 2.1.
No triangulo ABC, P e Q sao pontos no interior de ABC tais que 6 CBP = 6 P BQ = 6 QBA = 6 ABC/3
e 6 BCP = 6 P CQ = 6 QCA = 6 ACB/3. Sejam D e E as projecoes ortogonais de P sobre AB e AC,
respectivamente. Prove que AQ e perpendicular a DE.

Resolu
c
ao
Seja = 6 P AD. Ent
ao 6 AP D = 90 e, como
6
inscritvel. Logo AED = 6 AP D = 90 .
6

ADP e
6

AEP sao retos, o quadril


atero ADP E e

q
q
Q

90 - q

90 - q
a

b
b
b

Olhando a figura, note que basta provarmos que 6 QAC = . A e que entram os conjugados isogonais.
Como 6 P BC = 6 QBA e 6 BCP = 6 QCA, os pares de retas BP ; BQ e CP ; CQ sao simetricos entre si
em relacao `as bissetrizes de 6 ABC e 6 ACB, respectivamente. Ou seja, P e Q sao conjugados isogonais
e, portanto, 6 P AB e 6 QAC tambem sao iguais. Logo 6 QAC = e o angulo entre as retas AQ e DE e
180 (90 ) = 90 .
Note que para provar o resultado na conta, bastaria repetir a demonstracao do teorema fundamental
dos conjugados isogonais. Mas o mais interessante e que, sabendo da existencia dos conjugados isogonais, e
natural pensar nessa solucao. Em contraste, fazer a conta sem pensar em conjugados isogonais n
ao parece
ser t
ao natural assim. Ent
ao d
a para pensar que os conjugados isogonais nos economizou n
ao so fazer a
conta, mas mostrou onde fazer as contas relevantes.
2.2.

Conjugados isogonais dos pontos not


aveis

Voce ja deve estar familirializado com os pontos notaveis do triangulo: o baricentro (encontro das medianas),
o incentro (encontro das bissetrizes internas), o ortocentro (encontro das alturas) e o circuncentro (encontro
das mediatrizes). Quais sao os conjugados isogonais desses pontos? Vamos aproveitar e conhecer mais um
ponto notavel (mas n
ao t
ao conhecido).
Vamos fazer isso em ordem de dificuldade.
Incentro
As projecoes coincidem com as proprias bissetrizes. Logo o conjugado isogonal do incentro, que e o
encontro das bissetrizes internas, e ele mesmo.
O mesmo vale para os ex-incentros (encontros de duas bissetrizes externas e uma bissetriz interna e
centros dos ex-incrculos, que sao tangentes externamente aos lados ou seus prolongamentos). Pense sobre o
assunto!

Ortocentro e circuncentro
A figura a seguir deve convence-lo de que o ortocentro e o circuncentro sao conjugados isogonais.

Baricentro
Os isogonais das medianas sao as simedianas (SImetrico + MEDIANA). O ponto de encontro das
simedianas e o ponto de Lemoine, tambem conhecido como ponto simediano. O ponto de Lemoine e costumeiramente denotado por K.
Primeiro, vamos aprender a traca-las de modo mais pratico.
Lema. Sejam D a intersecao das retas tangentes ao circuncrculo do triangulo ABC por B e C. Ent
ao a
reta AD contem a simediana que passa por A.
Demonstra
c
ao

Construa o paralelogramo ABEC. Ent


ao AD contem a mediana AM . Afirmamos que D e E sao
conjugados isogonais. De fato, 6 BCE = 6 B e o angulo entre AC e CD, pela tangencia, e igual a 6 B.
Assim, as retas CD e CE sao conjugadas isogonais. Analogamente, BD e BE tambem sao, e o resultado
segue do teorema fundamental dos conjugados isogonais.
Exerccios
05. Sejam P e Q pontos no interior do
angulo 6 BAC tais que BP = CP , BQ = CQ e 6 ABP + 6 ACQ =
180 . Prove that 6 BAP = 6 CAQ.
06. As retas obtidas atraves das reflex
oes da diagonal BD do quadril
atero ABCD em relacao `as bissetrizes
de 6 B e 6 D passam pelo ponto medio de AC. Prove que as reflex
oes da diagonal AC do quadril
atero ABCD
em relacao `as bissetrizes de 6 A e 6 C passam pelo ponto medio de BD.
07. (Prova de Selecao EUA, 2008) Seja ABC um triangulo e G o seu baricentro. O ponto P varia sobre
o segmento BC. Os pontos Q e R pertencem aos lados AC e AB respectivamente, e sao tais que P Q e
paralelo a AB e P R e paralelo a AC. Prove que, ao variar P sobre BC, o circuncrculo de AQR passa por
um ponto fixado X tal que 6 BAG = 6 CAX.

08. (IMO 2004, Problema 5) Num quadril


atero convexo ABCD a diagonal BD n
ao e bissetriz do angulo
ABC nem do
angulo 6 CDA. Um ponto P no interior de ABCD satisfaz
6

P BC = 6 DBA

e
6

P DC = 6 BDA.

Prove que os vertices do quadril


atero ABCD pertencem a uma mesma circunferencia se, e somente se,
AP = CP .

3.

Tri
angulo pedal

Defini
c
ao 3.1. Seja P um ponto no plano do triangulo ABC e D, E e F as projecoes de P sobre as retas
BC, CA e AB. O triangulo DEF e o triangulo pedal de P em relacao ao triangulo ABC.
O que triangulos pedais tem de especial? Primeiro, aparecem muitos angulos retos, o que propicia o
aparecimento de quadril
ateros inscritveis. Segundo, eles normalmente minimizam areas.
Teorema do mnimo. Dado um triangulo T , considere todos os triangulos DEF semelhantes a T , todos
na mesma ordem, com D sobre o lado BC, E sobre o lado CA e F sobre o lado AB. Dentre todos esses
triangulos, o de menor
area e o triangulo pedal de algum ponto P .

Demonstra
c
ao
N
ao provaremos aqui a existencia de um triangulo de area mnima (caso voce esteja curioso, estude topologia
e depois volte!).
Seja DEF o triangulo de
area mnima. Seja M o ponto de Miquel de ABC e DEF , e sejam P , Q e R

as projecoes de M sobre os lados.

Note que o quadril


atero CP M Q e inscritvel (pois 6 M P C e 6 M QC sao retos), de modo que 6 DM E =
P M Q = 180 6 C. Portanto, 6 P M D = 6 QM E: imagine o angulo 6 DM E girando em torno de M para
coincidir com 6 P M Q; M D vira M P e M E vira M Q. Analogamente, 6 RM F = 6 QM E.
6

Portanto os triangulos P M D, QM E e RM F sao semelhantes e induzem uma roto-homotetia (voce se


MP
lembra o que e isso?) que leva DEF a P QR. A raz
ao de homotetia e MD
1, de modo que a area de P QR
e menor ou igual `
a
area de DEF . Como DEF tem area mnima, os triangulos devem ser consgruentes e
deste modo M P = M D, ou seja, P = D. Analogamente, Q = E e R = F , de modo que DEF e o triangulo
pedal de P .
Exemplo 3.1.
(Prova de Selecao EUA, 2008) Sejam P , Q, R pontos sobre os lados BC, CA, AB de um triangulo acut
angulo
ABC tais que P QR e equilatero e tem
area mnima entre todos tais triangulos equilateros. Prove que a reta
perpendicular a QR que passa por A, a reta perpendicular a RP que passa por B e a reta perpendicular a
P Q que passa por C tem um ponto comum.
Resolu
c
ao
Pelo teorema do mnimo, P QR e triangulo pedal de algum ponto T .

Como os
angulos 6 T QA e 6 T RA sao ambos retos, o quadril
atero AQT R e inscritvel, e o seu circuncentro
e o ponto medio A de AT . Assim, a reta perpendicular a QR e que passa por A, que contem a altura relativa

a QR, e isogonal a AT , que contem o circuncentro, em relacao ao triangulo AQR. Como os angulos 6 BAC
e 6 QAR sao iguais, a perpendicular e AT sao isogonais em relacao ao triangulo ABC tambem. O analogo
para as perpendiculares a P R por B e a P Q por C. Como AT , BT e CT sao concorrentes em T , seus
isogonais sao concorrentes no conjugado isogonal de T .
A ttulo de curiosidade, o ponto T e o primeiro ponto isodinamico. Os dois pontos isodinamicos (adivinhe
o nome do outro ponto!) sao os pontos de intersecao dos crculos de Apolonio de A, B e C (que passam
pelos vertices, o pe da bissetriz interna e tem centro sobre o lado oposto). Os seus conjugados isogonais sao
os pontos de Fermat. O primeiro ponto de Fermat e o ponto cuja soma das distancias aos vertices e mnima
(supondo que os
angulos internos do triangulos sao todos menores do que 120 ). Veja [5] para aprender isso
e muito, muito mais.
3.1.

Voltando `
as simedianas

Uma aplicacao interessante da ideia de triangulo pedal est


a relacionado `as simedianas. Uma outra maneira
de construir as simedianas e a seguinte:
Lema. Construa quadrados ABBc Ac , BCCa Ba e CAAb Cb externamente sobre os lados do triangulo ABC.
Prolongue Ac Bc , Ba Ca e Cb Ab para obter o triangulo A B C . Ent
ao as retas AA , BB e CC concorrem
no ponto simediano K de ABC.

Demonstra
c
ao
Por simplicidade, sejam BC = a, CA = b e AB = c e L o encontro de AA , BB , CC . Queremos provar
que L = K.
Primeiro, como os pares de retas AB; A B , BC; B C e CA; C A sao paralelos, os triangulos ABC e
A B C sao semelhantes. Seja k a raz
ao de semelhanca. Sejam ka , kb e kc as distancias de K a BC, CA e
AB, respectivamente. Das semelhancas entre KAB; KA B , KBC; KB C e KCA; KC A , todas de raz
ao
k,
kb
kc
ka
kb
kc
k
ka
=
=
= k
=
=
=
ka + a
kb + b
kc + c
a
b
c
1k

Isto quer dizer que as distancias de L a cada um dos lados e proporcional aos seus comprimentos. Alem
disso, considerando uma semelhanca prova-se que um ponto X pertence a, digamos, AL se, e somente, as
distancias de X aos lados AB e AC sao proporcionais a seus comprimentos. Basta provar que a simediana
por A tem a mesma propriedade. Para isso, considere a construcao anterior, sendo D o mesmo ponto definido

anteriormente.

Sendo x e y as distancias de D a AB e AC, respectivamente, considerando que o angulo entre AB e


BD e 6 F BD = 6 ACB = 6 C e o
angulo entre AC e CD e 6 DCE = 6 ABC = 6 B (nao se preocupe com
triangulos obtusangulo; nesse caso, troque o angulo obtuso por seu suplementar), nos triangulos ret
angulos
BDF e CDE, x = BD sen 6 C e y = DC sen 6 B. Observando ainda que, sendo DB e DC tangentes,
sen 6 C
AB
DB = DC, temos xy = sen
em. Da mesma
6 B = AC . Logo D pertence a AL e, consequentemente, K tamb
forma provamos que K pertence a BL e CL, de modo que L = K.
Assim como no teorema das bissetrizes, as simedianas dividem os lados opostos em raz
oes interessantes.
Lema. Seja ABC um triangulo e AN uma simediana. Ent
ao

BN
CN


AB 2
.
AC

Demonstra
c
ao

Ja provamos anteriormente que as distancias do ponto simediano K aos lados sao proporcionais a seus
comprimentos. Ent
ao existe t real tal que ka = ta, kb = tb e kc = tc. Assim, as areas de KAB, KAC e
tc2 tbb
tb2
taa
ta2
KBC sao tcc
=
2
2 , 2 = 2 e 2 = 2 , respectivamente. Logo
area ABN

area KBN

area ABN area KBN


area KAB
c2
BN
=
=
=
=
= 2 =
CN
rea ACN
a
rea KCN
a
area ACN area KCN
area KAC
b
Isso pode ser generalizado:

AB
AC

2

Lema. Sejam da , db e dc as distancias de um ponto P aos lados BC, CA e AB do triangulo ABC. Se AP


cdc
corta BC em N , ent
ao BN
CN = bdb .
Demonstra
c
ao
Fica a cargo do leitor.
3.2.

A desigualdade de Erd
os-Mordell

Um dos principais teoremas sobre triangulos pedais e a desigualdade de Erd


os-Mordell:
Desigualdade de Erd
os-Mordell. Seja P um ponto no plano do triangulo ABC e da , db , dc as distancias
de P `as retas BC, CA, AB respectivamente. Ent
ao
P A + P B + P C 2(da + db + dc )
Demonstra
c
ao
Seja P A = d. Multiplique a figura original por d e construa triangulos semelhantes aos triangulos obtidos
por P A e as projecoes de P sobre AB e AC:

Note que 6 GDE = 90 e 6 HDF = 90 , de modo que G, D e H sao colineares. Alem disso,
EGD e 6 EHD sao ambos retos, de modo que as retas EG e F H sao paralelas. A distancia entre essas
duas retas e GH = AB db + AC dc , que e menor ou igual a EF = d BC. Lembrando que d = P A, temos
AC
AB
db + BC
dc . Analogamente,
AB db + AC dc P A BC P A BC
6

AB
AC
db +
dc
BC
BC
BC
AB
da +
dc
PB
AC
AC
BC
AC
da +
db
PC
AB
AB
PA

Somando as tres equacoes e lembrando que t +


PA + PB + PC

AB
AC
+
AC
AB

da +

1
t

2 para todo t real positivo,

AB
BC
+
BC
AB

db +

AC
BC
+
BC
AC

dc 2(da + db + dc )

Exemplo 3.2.
(IMO 1991, Problema 4) Sejam ABC um triangulo e M um ponto interior. Mostre que pelo menos um dos
angulos 6 M AB, 6 M BC e 6 M CA e menor ou igual a 30 .
Resolu
c
ao
Sejam P , Q e R as projecoes de M sobre BC, CA e AB, respectivamente.

MR
,
Pela desigualdade de Erd
os-Mordell, M A + M B + M C 2(M P + M Q + M R). Se todas as raz
oes MA
1
sao maiores do que 2 , ent
ao M A < 2M R, M B < 2M P e M C < 2M Q, e M A + M B + M C <
e menor ou igual a 12 . Todavia,
2(M P + M Q + M R), contradicao. Ent
ao uma das raz
oes, digamos, MR
MA ,
MR

6
6
MA = sen M AB, de modo que M AB 30 .
MP MQ
MB , MC

Exerccios
09. Dado um triangulo com permetro L, seja P o permetro de um triangulo pedal. Prove que L 2P .
Quando ocorre a igualdade?
10. Seja P um ponto interior ao triangulo ABC e T o seu triangulo pedal. Prove que a area de T e igual
2
OP 2
a R 4R
vezes a
area de ABC.
2
11. Seja G o baricentro do triangulo ABC e D, E, F as projecoes ortogonais de G sobre os lados BC, CA
e AB, respectivamente. Prove que
4
area DEF
1
<

27
area ABC
4
12. Seja P um ponto qualquer no plano do triangulo ABC. As projecoes de P sobre BC, CA, AB sao D,
E e F respectivamente.
(a) Prove que as perpendiculares a EF , F D, DE por A, B, C respectivamente tem um ponto P em comum.
(b) Sejam Q e Q as segundas intersecoes de AP e AP com o circuncrculo de ABC, respectivamente.
Prove que as retas QQ e BC sao paralelas.
13. Os pontos X, Y e Z est
ao sobre BC, CA e AB do triangulo ABC, respectivamente, e sao tais que XY Z
e ABC sao semelhantes, nessa ordem. Prove que o circuncentro de XY Z e equidistante aos ortocentros de
ABC e XY Z.
14. (Ibero 2008, Problema 5) Seja ABC um triangulo e X, Y , Z pontos interiores dos lados BC, AC, AB,
respectivamente. Sejam A , B , C os circuncentros dos triangulos AZY , BXZ, CY X, respectivamente.
Demonstre que
(ABC)
(A B C )
4
e que a igualdade ocorre se, e somente se, as retas AA , BB , CC tem um ponto em comum.

Observacao: Para um triangulo qualquer RST , denotamos a sua area por (RST ).
15. (OPM, 2001)

C
z
v
x
A

u
M
w

y
B


(a) Na figura acima, considere pontos B1 e C1 sobre as semirretas AB e AC, respectivamente.
(i) Mostre que a soma das
areas dos paralelogramos com lados AB1 e AM e com lados AC1 e AM e
igual `
a
area do paralelogramo tal que um de seus lados e B1 C1 e o outro e paralelo a AM .
(ii) Tomando AB1 = AC e AC1 = AB, conclua que AB v + AC w BC x
(b) Prove a Desigualdade de Erd
os-Mordell: 2(u + v + w) x + y + z
4.

Refer
encias Bibliogr
aficas

[1] Uma otima fonte de problemas e o Mathlinks: http://www.mathlinks.ro/ (em ingles).


[2] Para quem gosta de Geometria, o Forum Geometricorum e um prato cheio! Tudo sobre quadril
ateros
completos foi retirado do artigo Steiners Theorems on The Complete Quadrilateral, de Jean-Pierre
Ehrmann, Volume 4 (2004), pp 3552.
[3] Para quem quer saber mais sobre o teorema de Erd
os-Mordell (e ver ainda mais uma demonstracao!),
veja o artigo de Anderson Torres, A Desigualdade de Erd
os-Mordell, na Eureka! 18.
[4] O livro Modern Geometry of the Triangle, de William Gallatly, contem muita informacao interessante,
incluindo a maior parte dos fatos sobre simedianas e o ponto simediano.
[5] Mais conjugados isogonais? Isso e muito mais no livro Geometry of Conics (o livro do bode veja a
capa!), de A. V. Akopyan e A. A. Zaslavsky.

Você também pode gostar